Process Of Examination Of Patent Application Question and Answers

Process Of Examination Of Patent Application

Question 1. Company XYZ Ltd. has made an invention on a new water purifier technology and the company has filed a patent application. The company fears that the Public use of that invention before the date of filing of the application destroy the novelty of the invention. Convince the company XYZ Ltd. why they should not fear by illustrating cases in India.
Answer: Prior public use of the invention before the date of filing of the application destroys the novelty of the invention. However, there is an exception to this general rule.

The Act provides that if an invention has been publicly worked in India within one year before the priority date by the patentee or applicant for the patent or by any third person from whom he derives the title or by the person who has obtained consent to work the invention and such working of the invention was only for reasonable trial and it was necessary to effect such trial or working in public given the nature of the invention then such working of the invention does not anticipate the invention.

The Indian patent law takes the middle way and provides for grace periods in some conditions to evaluate anticipation.

The Indian Patents Act mentions what are not anticipations in Sections 29 to 34, rather than defining anticipation.

Exceptions are mentioned in the Indian Patent Act under which the patent application can be filed despite public disclosure, and such public disclosure will not be considered to have been anticipated.

To conclude, one should file a patent application ideally before publicly disclosing the invention. However, in light of the provisions discussed above one can still contemplate patent application filing.

Lack of novelty is usually referred to as ‘anticipation and is determined by factors such as prior publication, public knowledge, and public use. Commercialized products and selection inventions.

While anticipation is not expressly defined in the Patent Act, Sections 29 to 34 identify what anticipation is not. When testing for anticipation, if a prior art exists (i.e. if the prior art describes something falling within the scope of an alleged claim then, if by studying the prior art the claimed invention can be performed, the claim would be considered to be anticipated.

It may be unnecessary to repeat the prior art test, but expert opinion may be considered to identify anticipation better using relevant expertise.

It can also be identified by showing the predictable result as the outcome of what is described in the prior art, regardless of whether it would be a product or process falling within the scope of the claim.

In India, a patent application is considered to be anticipated if the invention is disclosed in a patent or any other document that is published before the priority date of the application.

However, if the inventor proves that the matter published was obtained fraudulently and was published without his or her consent, then it may not be considered to be anticipated. The concept for identifying prior publication was established in Farbewerke Hoechst Aktiengesellschaft Vormuls. Meister Lucius v.

Unichem Laboratories, wherein the court held as follows: “To anticipate a patent, a prior publication or activity must contain the whole of the invention impugned; i.e., all the features by which the particular claim attacked is limited.

In other words, the anticipation must be such as to describe, or be an infringement of the claim attacked.”

Likewise, in Lailubhai Chakubhai Jariwala v. Chimanlal Chunilai and Co., the court observed that:

‘The two features necessary to the validity of a patent are novelty and utility, but the real test is the novelty of the invention.

Novelty is essential, for otherwise there would be no benefit given to the public and consequently, no consideration moving from the patentee [while interpreting the factor related to public knowledge and public use.”

The court further held that “the next question is, whether the plaintiff’s invention has been anticipated by prior public user.

Has it been publicly used by the plaintiff and/or by others before the date of the application? Public user does not mean a user or exercise of the invention by the public, but a user or exercise in a public manner; and it is in every case a question of fact, If the invention is being put into practice before and at the date of the grant, the grant will not be for a new invention or manufacture, and this applies equally whether the invention is being practiced by the patentee himself or by others.

A use of the invention for trade may constitute a prior user which invalidates the patent, and it has been held that the prior public sale of goods or articles treated according to the invention is a public user of the invention, for the sale is strong evidence that the user was commercial and not experimental”.

In Monsanto Co. v. CoromandelIndag Products (P) Ltd. 1986 A.I.R. 712, it was held that “to satisfy the requirement of being publicly known as used in clauses (e) and (f) of Section 64(1 ), it doesn’t need to widely be used to the knowledge of the consumer public.

It is sufficient if it is known to the persons who are engaged in the pursuit of knowledge of the patented product or process either as men of science or men of commerce or consumers.

Question 2. Write a brief note on the novelty of the invention.
Answer: The novelty of Invention:

  1. An invention is considered new (novel) if it has not been anticipated by publication in any document anywhere in the world, or prior claimed in an application for patent in India, or forms part of the knowledge, oral or otherwise, available within any local or indigenous community in India or elsewhere, or used;
  2. Before the date of filing of patent application or date of priority, whichever is earlier, that is, the subject matter has not fallen in the public domain or that it does not form part of the state of the art.
    • The following are the general principles relating assessment of Novelty:
    • An invention is considered new if it is not anticipated by prior publication, prior use, or prior public knowledge.
    • An invention is new (novel) if it has not been disclosed in the prior art, where the prior art means everything that has been published, presented, or otherwise disclosed to the public before the date of filing of the complete specification.
    • To determine novelty, an application for a Patent filed at the Indian Patent Office before the date of filing of the complete specification of an inter-filed application but published after the same is considered for prior claiming.
    • While ascertaining novelty, the Examiner takes into consideration, inter alia. the following documents:
  3. Which have been published before the date of filing of complete specification.
    1. Such Indian Patent Applications which have been filed before the date of filing of the complete specification and published on or after the date of filing of the complete specification, but claim the same subject matter.
    2. Also, the Examiner may consider such documents that have been published before in a transaction of a learned society or exhibited before in an authorized manner as designated by the Government within one year from the date of such filing.
  4. A prior art will be considered anticipatory if all the features of the invention under examination are present in the cited prior art.
  5. The prior art should disclose the invention either explicitly or implicitly.
  6. The Mosaic king of prior art documents is not followed in the determination of novelty.
  7. A generic disclosure in the prior art may not necessarily take away the novelty of a specific disclosure.
  8. A specific disclosure in the prior art takes away the novelty of a generic disclosure.
  9. In a case where a prior art is cited as an anticipation in the Examination Report, which is not deemed to be an anticipation by reason on Section 29-34, the onus of proving is on the applicant.

Question 3. Discuss the concept of patent application.
Answer: Publication of Patent Application: Usually a patent application is published in the Official Patent Office Journal after the lapse of 18 months from the date of filing of the application or the priority claimed date, whichever is earlier.

This publication includes all pertinent details related to the application. It includes the title, abstract, application number, and name and address of the applicant. After this publication, a patent application becomes open for public scrutiny.

An exciting concept of request for early patent publication is also available for the applicant. This can be done when the applicant wants his application to be published before the normal period of 18 months.

Early publication stands for making a patent public before the time of its normal publication. This could be of help when one is planning to sell or license the patent or seek investor and related advantages.

An early publication of an application is allowed as per Section 11(A)(2) of the Indian Patents Act, on payment of the prescribed fee.

Section 11(A)(2) of the Indian Patents Act mentions: The applicant may, in the prescribed manner, request the Controller to publish his application at any time before the expiry of the period prescribed under sub-section (1) and subject to the provisions of sub-section (3), the Controller shall publish such application as soon as possible.

Once such a request is made, the patent office has to publish the application ordinarily within one month from the date of request for publication.

The applicant may request the controller for early publication of the patent application before the lapse of 18 months. Such a request has to be made on Form-9- request for early publication.

Request for Examination: The Patents Act, 1970 provides for the examination of patent applications only on filing of a request for examination by the applicant or any other interested person [section 11 B], This request can be filed on Form-18 with a prescribed fee at any time within 48 months from the date of priority or from the date of filing of the application, whichever is earlier.

The patent application is referred to the examiner strictly in order of the requests filed.

The examiner to whom the application is referred for examination has to submit his report to the Controller ordinarily within one month from such reference but not exceeding three months from such reference.

Allocation of Application to Examiner for Examination: Once the request for examination is received and the application has been published, the Controller shall refer the particular application to an examiner for conducting examination and search under sections 12 and 13 of the Patents Act, 1970.

Before such a reference, the controller has to consider the following points. In order of filing of request: Reference of the patent application shall be strictly per the sequential order of filing of the request for examination.

Examination of Patent Application:

Regulatory Regime: The examination of patent applications is conducted under the provisions of section 121 of the Patents Act, 1970.

After the patent application is filed and after the filing of the request for examination as well as the publication of the same, the Controller shall refer the application and the specification and other documents related thereto to an examiner for making a report to him under the provisions of the Act and the rules made there under.

Question 4. What do you mean by examination of a patent application and discuss the different types of examination of a patent application?
Answer:

Formal examination: The patent examination can broadly be classified into two distinct forms, the formality examination and the substantive examination.

The following steps are involved in the formal examination of patent applications:

  1. To check whether the application, specification, and other related documents are filed in duplicate in prescribed forms or not.
  2. To check whether the applicant is entitled to apply for a patent under section 6 of the Act.
  3. To check the jurisdiction of the applicant as specified under Rule 4 of the Patents.
  4. Rules to decide the Appropriate Office for processing of patent application.
  5. Jurisdiction is normally decided on the normal residential or domiciled address or place of business of the applicant or the First Mentioned Applicant, in case of joint applicants or the place from where the invention originated.
  6. To check the jurisdiction of the applicant who has no place of business or domicile in India.
  7. The address for service in India, as given by the applicant, is to be taken into consideration for deciding the Appropriate Office.
  8. To check whether the address for service has been provided in the application. If not, the Controller has no obligation to proceed further (the Controller may take a suo-moto decision in the matter)
  9. To check whether any request has been made for post-dating of the provisional specification.
  10. Postdating is allowed for a maximum period of 6 months (Sec -17(1)).
  11. To check whether the complete specification is filed within 12 months from the date of filing of provisional specification as specified in section 9(1) of the Act.
  12. The 12 months for filing the complete specification after the provisional specification is not extendable.
  13. To check whether the complete specification is filed within 12 months from the earliest provisional specification when the same applicant has filed more than one provisional specification in respect of inventions which are cognate or of which one is a modification of the another and the whole of such inventions are such as to constitute a single invention(Sec -9(2)).
  14. To check whether the complete specification is filed within 12 months from the earlier complete specification filed which was treated as provisional specification under the provisions of section 9 (3) of the Act.
  15. It is to be noted that there is no provision for filing provisional specifications or making a request to convert the complete specification to provisional specifications in respect of the applications filed under convention and national phase entry via the PCT system.
  16. To check whether a Power of Attorney or a General Power of Attorney in the original has been filed and whether the patent agent is authorized to practice before the patent office on behalf of the applicant(s).
  17. Self-attested photocopy of a General Power of Attorney is also admissible provided, an indication that the earlier patent application with which the original GPA is attached, has been submitted.
  18. To check whether Declaration as to Inventorship (Form -5) has been filed along with the specification filed after filing provisional specification or along with the complete specification filed under convention application or along with the complete specification filed as PCTNP application under PCT route, as the case may be.
  19. To check whether Proof of Right to make an application has been filed as specified in Section 7(2) of the Patents Act along with the application (even at the time of filing a provisional application) except in the cases where the inventor(s) is(are) applicant(s) by himself (themselves).
  20. To check whether Form 3 has been filed along with the patent application or within a period as specified under section 8 of the Patents Act.
  21. To check whether the application has been published under the provisions of Section 11 A If the application is published before the period of 18 months from the date of applying, a check has to be made whether the request in Form – 9 has been filed for early publication, along with the requisite fee and, whether the application has been published after taking Form- 9 on record.
  22. To make cross-reference (s), if any, on the file covers of co-pending applications (cognate type, divisional, and parent applications) The related applications shall be sent together physically to examiners.
  23. A check is to be made whether the request for examination (Form- 1 8) has been filed along with the requisite fee and by whom it was filed.
  24. If Form 18 has been filed by a person other than the applicant it is to be examined whether that person is the ‘person interested’ as defined in Section 2(1)(t) of the Patents Act.
  25. It also needs to be checked as to how many priorities are claimed and whether the requisite fee has been paid or not. Timelines of filing documents and RQs, Forms and fees, right to file, priority rights, etc.
  26. The timeline as provided in the Act and Rules has been suitably incorporated in Annexure-I.

Substantive Examination: The examiners to whom the application is referred under section 12 examines the patent application together with the complete specification and the other documents related there for making a report in respect of matters as mentioned in section 12(1 )[(a) to(d)] to the Controller.

The examiner determines whether any lawful ground of objections exists to the grant of a patent under the statute.

Question 5. What are single inventive concepts?
Answer: Single Inventive Concept: Section 10(5) mandates that the claim/ claims of the complete specification shall relate to a single invention, or to a group of inventions linked to form a single inventive concept.

The Manual of Patent Office Practice and Procedure, of year 201 6 allows that there may be more than one independent claim in a single application if the claims fall under a single inventive concept.

In the Manual, it has been advised “While there is no restriction as to the number of claims, including independent claims, it is advisable to limit the number of claims, as well as the number of independent claims in a single application so that the claims are linked to form a single inventive concept.

If claims relate to a plurality of distinct inventions, it may be objected on the ground of lack of unity of invention”. The single common technical relationship which is inventive is called the “special technical feature”.

This determination should be based on the content of the claims supported by the description in the light of the prior art.

Unity of invention is present only when there is a “technical relationship” among the claimed inventions involving one or more of the same or corresponding “special technical features.”

The expression “special technical features” means those technical features that define a contribution that each of the claimed inventions, considered as a whole, makes over the prior art.

The determination of whether a group of inventions is so linked as to form a single inventive concept is made without regard to whether the inventions are claimed in separate claims or as alternatives within a single claim.

Lack of unity may be evident in an application in the following ways:

‘A priori’, i.e., before consideration of prior art, if the claims falling in different groups do not share the same or corresponding technical feature.

‘A posteriori’, i.e., after a search of the prior art, if the shared technical feature fails to make an inventive contribution over the prior art.

Lack of unity of invention may be directly evident “a priori,” that is, before considering the claims with any prior art, or may only become apparent “a posteriori,” that is, after considering the prior art.

Question 6. Discuss the re-issue and re-examination.
Answer: Re-issue and Re-Examination: After the grant of a patent, every patentee has to maintain the patent by paying a renewal fee every year as prescribed in Schedule I. For the first two years, there is no renewal fee.

The renewal fee is payable from 3rd year onwards. In case the renewal fee is not paid the patent will be ceased. To keep a patent in force renewal fees are payable at the expiration of the second year from the date of the patent or of any succeeding year.

In other words, the renewal fee has to be every year up to the completion of 20 years. Renewal fees can be paid beyond the due date within a period of 6 prescribed fees.

If a patent is granted later than two years from the date of filing of the application, the fees which have become due in the meantime may be made within 3 months from the date of recording the patent in the register.

This time is also extendable by 6 months as described earlier. The patentee has the choice to pay the renewal fees every year or he can pay in lump sum as well.

Further, a request for restoration of the patent can be filed within 18 months from the date of cessation of the patent along with the prescribed fee. After receipt of the request, the matter is notified in the official journal for further processing of the request.

Preparation Of Patent Documents Question and Answers

Preparation Of Patent Documents

Question 1. Read the following case on patent law and answer the questions that follow:
Answer: On 7th July 2005, Viraj Ltd. (Viraj) was granted a patent under Indian Patent No. 195904, with a priority date of 16th July 2002. The title of the patent application was “an improved internal combustion engine working on four-stroke principle.”

The invention was called DTS-i Technology and it related to the use of twin spark plugs located diametrically opposite to each other in a small displacement engine with the cylinder bore diameter ranging between 45 mm to 70 mm.

According to Viraj, this placement of the spark plugs enabled better control over the ignition timing, and less time was taken for the flame to travel during the process of combustion.

The novelty also lay in the use of a sleeve to protect the spark plug which prevented exposure of the plug to the lubricating oil.

In 2003, Viraj launched ‘Inventa’, a motorcycle that employed the DTS-i Technology in respect of which the patent was then pending.

In the first eight months of that financial year itself, Viraj manufactured and marketed 8,1 4,393 two-wheelers with the DTS-2 Technology out of a total of 1 5,01,241 two-wheelers which were sold by it amounting to 54.25% of its total sales.

In 2007, CVS Ltd. (CVS) announced the launch of a 125-cc motorcycle under the trademark ‘Sport’ which was to be powered by a lean burn internal combustion engine of bore size 54.5 mm with a twin spark plug configuration just like that of Viraj. CVS also stated that on the 1st and 3rd of September, 2007, Viraj had issued certain groundless threats to dissuade CVS from launching ‘Sport’.

Hence, in October 2007, CVS filed a suit under sections 105 and 106 of the Patents Act, 1 970 in the High Court, alleging that the statements made by Viraj on the 1st and 3rd of September, 2007 constituted groundless threats, and sought the intervention of the Court to restrain Viraj from interfering with the launch of ‘Sport’.

Further, CVS also applied for the revocation of Viraj’s patent before the Indian Patents Appellate Board under section 64 of the Patents Act, 1970.

Upon the announcement by CVS, Viraj filed a suit for permanent injunction under section 108 of the Patents Act, 1970 in the High Court to restrain CVS from using the internal combustion technology patented by Viraj and from employing the same in marketing 2-3 wheelers, including CVS’s proposed 1 25-cc ‘Sport’ motorcycle.

Vide its order dated 16th February 2008, the Single Judge of the High Court restrained CVS from launching the proposed 1 25-cc ‘Sport’ motorcycle with the twin spark plug engine technology, as Virajprima facie enjoyed the right of exclusive usage of the patent, granted to it by the Patent Office.

The High Court held that Viraj had succeeded in establishing a prima facie case for the grant of an injunction, and while granting the injunction was pleased to observe:

“…..Suffice it to say now at this stage, prima facie there is novelty which means an invention and the same has been registered under the Patents Act…. novelty has been on the face of it proved by the applicant by marketing the product to such a large extent and also without objection fairly for long ….”

Aggrieved by the order of the Single Judge, CVS preferred an appeal before the Division Bench of the High Court. The Appellate Bench of the High Court held that Viraj had not succeeded in establishing a ‘prima facie’ case of infringement in respect of its patented twin spark technology.

The Division Bench observed that having regard to the nature of operation of the DTS-i Technology engine by twin spark plugs and that of CVS by receipt of air-fuel mixture through two different intake valves, their points of emphasis differed considerably, notwithstanding the use of twin spark plug in both the technologies.

The Division Bench further observed that the operation of the invention as claimed by Viraj appears to be plug-centric and that of CVS was valve-centric, and on scrutiny of the claim as set out in the final complete specification,

It held that it found it difficult to countenance Viraj’s claim that CVS’s product specification infringed Viraj’s patented right. Accordingly, the Division Bench set aside the order of the Single Judge.

Aggrieved by the order of the Division Bench, Viraj preferred an appeal before the Supreme Court.

The Supreme Court, while prima facie agreeing with the order of the Division Bench, ordered that although CVS shall be entitled to sell its motorcycle ‘Sport’, it shall maintain an accurate record of its entire domestic and international sale and directed the High Court to appoint a receiver in this connection.

In its interim order, the Supreme Court reiterated that in matters relating to trademarks, copyrights, and patents, the provisions of the Code of Civil Procedure, 1908 which mandate that civil disputes should be heard on a day-to-day basis without any adjournments, except in circumstances beyond the control of the parties.

It also directed that the final judgment should be given normally within four months from the date of the filing of the suit.

The Supreme Court directed that the timeline stipulated above be adhered to ‘punctually and faithfully’ by all courts and tribunals in the country.

Question 2. What were the grounds under which the Division Bench of the High Court reversed the decision of the Single Judge of the High Court? Cite relevant case law.
Answer: The Division Bench of the High Court reversed the decision of the Single Judge of the High Court on the following grounds:

  1. After considering the functioning and performance of products manufactured by both parties, the Division Bench found that both the products had distinctive features and different operations.
  2. As to the nature of the operation of the engine, it was observed that the DTS-i Technology engine is operated by spark plugs, and the CVS engine is operated by receipt of air-fuel mixture through two different intake valves.
  3. The technology used by the appellant was different from the technology used by the respondent.
  4. The point of emphasis of both the engines is considerably different and does not bear an identified resemblance even with the use of twin spark plugs in both technologies.
  5. It is further observed that Viraj’s engine is plug-centric and CVS’s engine is value-centric.
  6. On the scrutiny of the final complete specification, it was found that it is difficult to allow Viraj’s claim that CVS’s product specification has infringed Viraj’s patented rights.
  7. Along with this, the distinction between a patented claim and the infringed product is well protected under provisions of the Patents Act, of 1970. Hence, infringement of the respondent’s patent by the appellant has not been proved.

 

Question 3. Elaborate on the order and the directions passed by the Supreme Court and how it may impact the IPR pending litigation.
Answer: Challenging the appeal allowed by the Division Bench of the Madras High Court, Viraj appealed to the Supreme Court by special leave.

The most salutary effect of this case has been the guidelines passed by the Supreme Court directing expeditious disposal of IPR cases by the Trial Court.

In this case, the Court thought that the matters relating to trademarks, copyrights, and patents should be decided expeditiously and finally by the Trial Court instead of merely granting or refusing to grant an injunction.

Experience shows that in the matters of trademarks, copyrights, and patents, litigation is mainly fought between the parties about the temporary injunction which goes on for years and the result is that the suit is hardly decided finally.

This is not proper. The Court observed that once the hearing of the suit has commenced, it shall be continued from day to day until all the witnesses in attendance have been examined, unless the Court finds that, for exceptional reasons to be recorded by it, an adjournment of the hearing beyond the following day is necessary.

Only circumstances which are beyond the control of the parties would justify such an adjournment.

It also opined that in suits relating to the matters of patents, trademarks, and copyrights, final judgment should be given normally within four months from the date of filing of the suit.

The direction of the Supreme Court seeks to enforce a provision under which, an adjournment on the ground that the pleader of a party is engaged in another Court, shall not constitute a ground for an adjournment.

This cause of adjournments is the biggest bane and obstacle in overcoming judicial delays, and if these guidelines are implemented by the Courts and Tribunal below, all IPR cases would be decided expeditiously. This augurs well for strengthening the rights of IPR holders in India.

The relevant Case Law is M/s TVS Motor Company Limited Vs. M/S Bajaj Auto Limited (MANU/SC/1632/2009).

Question 4. Explain the factors which constituted a prima facie case for the grant of an injunction.
Answer:  For an invention to be patentable, it must satisfy the triple tests of novelty, non-obviousness, and capability of industrial application and exploitation.

While considering the facts of the case, the Court found that the invention satisfied the triple tests and was patentable.

The Single Judge order of the High Court emphasised that in cases of interlocutory injunction, including any patent action the plaintiff must prove or show as follows:

  1. Prima facie case that the patent is valid and infringed.
  2. The balance of convenience lies in favor of the plaintiff and
  3. Irreparable loss would be caused to the plaintiff by not granting an order of injunction.

The factors which constitute a prima facie case for grant of an injunction are as follows:

  1. Firstly, whether the applicant holds a patent over the technology under consideration. The certificate for the grant of a patent by the appropriate authority in such cases is prima facie proof of the grant of a patent.
  2. Secondly, whether there is a commercial use of the aforementioned patented technology by the respondent/non-applicant.

In cases where there is a dispute between the parties as to whether the invention or technology claimed to be patented is patentable or not the tests applied by the Courts are the test of novelty non-obviousness, and capability of industrial application and exploitation of the technology.

In the relevant case law cited i.e. Bajaj Auto Limited Vs. TVS Motor Company Limited, while considering the facts of the case, the Court found that the invention satisfied the triple tests and was patentable.

The Court was of the view that, upon the amendment of the Act which came into force on 20 May 2003, when a patentee files a suit for infringement based on the patent granted to him, the patent should be prima facie presumed to be valid, until the same is revoked or set aside under any of the grounds set out in the Act or any other manner.

The Court was of the view that before the amendment, the patentee was conferred an exclusive right to use himself or through agents or licensees and also to exercise sell, or distribute the inventions in India.

However, after the amendment, the patentee now enjoys an exclusive right to prevent third parties from using selling, or offering for sale, etc. his invention.

On the question of whether Bajaj’s invention involved an inventive step, the Court was satisfied that the invention involved a technical advance as compared to the existing knowledge and that it had economic significance.

The Court was prima facie satisfied that the invention had found a special place in the Indian market and had established a significant market share.

The Court found that even though Bajaj’s product ‘Pulsar1 had been introduced in the market in 2003, TVS had for the first time objected only on August 24, 2007, by which time Bajaj’s DTS-i technology-based product had been sold in large numbers across the country.

The Single Judge of the Madras High Court found that the petition for revocation of the Patent granted to Bajaj had been filed a mere six days before the launch of Flame, and as such the conduct of TVS was not entirely bona fide.

The Court was of the view that till such time, TVS succeeded in its petition for revocation, the patent granted to Bajaj could not be viewed with suspicion, considering that it had been in existence for more than five years and as such the patentee Bajaj must be treated as an actual user and the presumption of the validity of its patent was thus established.

On the above basis, the Court recorded its satisfaction that Bajaj had succeeded in establishing a prima facie case for the grant of an injunction.

The Court also recorded that having exploited the patent for five years, the patentee had a further period of fifteen years to exploit the patent, and hence, the balance of convenience was also in its favor. Accordingly, the Madras High Court granted an interlocutory injunction in favor of Bajaj.

Question 5.
Write a note on the significance of laboratory notebooks/logbooks/record books in patent litigation.
Answer: Laboratory notebooks are the birthplace of inventions. Laboratory notebooks (also called journals, inventor’s notebooks, or log books) are used by inventors, scientists, and engineers to record their invention process, experimental tests, ideas, results, and observations.

It is not a legal document but is valuable if properly organized and maintained since it can help establish dates of conception and reduction to practice.

The purpose of the inventor’s logbook is simply to be an irrefutable diary of your progress, so that if it comes to a legal conflict, you can produce your logbook and some supporting evidence to substantiate your claim that you were the first to start working on your invention, and the first to “make a working model” of your invention.

The information can improve the outcome of a patent or a patent contestation. Typically, governments award patents on either a first-to-file or first-to-invent basis.

Therefore, it is important to keep and maintain records that help to establish who is first to invent a particular invention.

A laboratory notebook is a systematic device for recording all information related to an invention in such a way that it can be used as a key component to develop a case during a patent contestation or patent-related lawsuit.

When properly kept, the notebook is a valuable tool for the inventor since it provides a chronological record of an invention and its reduction to practice. Each entry must be signed and dated by a witness.

The witness should not be someone with a conflict of interest (such as a research partner).

If an inventor ever has to go to Court to prove that he or she was the first to invent, then the witness would be called to the stand to testify that it is their signature and they have signed that page on that date.

Question 6. Discuss the typical parts of a patent application.
Answer: A patent application is a request pending at a patent office for the grant of a patent for the invention described and claimed by that application.

An application consists of a description of the invention (the patent specification), together with official forms and correspondence relating to the application.

The term patent application is also used to refer to the process of applying for a patent or to the patent specification itself.

The typical parts of a patent application are generally:

  1. Claims
  2. Detailed description (or specification)
  3. Drawings
  4. Background
  5. Abstract
  6. Summary

The title should broadly describe the invention. However, titles are not generally examined. Occasionally a patent examiner decides that a title is not descriptive of the invention.

It is best to avoid being overly narrow in the invention’s title, although the title should sufficiently indicate the subject matter of the invention. A patent application should also include the names of the inventors.

The inventors should be named after the title, e.g. on the cover page. The patent application itself should also include all priority information, such as the identification of related applications.

It is desirable to remember who the audience will be for the patent application. The key audiences include judges and patent examiners.

Of course, the patent agent’s client and the inventor are also audiences; the patent agent must make sure the inventor understands his patent application.

Other potential audiences include competitors, infringers, and investors.

Many investors will often scrutinize a technology company’s patent portfolio before investing.

Question 7. What is a Laboratory Notebook?
Answer: Laboratory notebook is also known as Log books, Record books, journals, and inventor’s notebooks. It is the birthplace of inventions.

A laboratory notebook is a systematic device for recording all information related to an invention in such a way that it can be used as a key component to develop a case during a patent contestation or patent-related lawsuit.

When properly kept, the notebook is a valuable tool for the inventor since it provides a chronological record of an invention and its reduction to practice. Each entry must be signed and dated by a witness.

If an inventor ever has to go to Court to prove that he or she was the first to invent, then the witness would be called to the stand to testify that the signature is theirs and they signed that page on that date.

The uses of laboratory notebooks are:-

  • It is used by inventors, scientists, and engineers to record their inventions, processes, experimental tests, ideas and results, and observations.
  • It is not a legal document but is valuable if properly organized and maintained since it helps establish dates of conception and reduction to practice.

Question 8. How a patent specification is prepared? Discuss the types of patent specifications.
Answer:

  • A patent specification can be prepared by the applicant himself or his registered and authorized agent.
  • The patent specification generally comprises the title of the invention indicating its technical field, prior art, drawbacks in the prior art, the solution provided by the inventor to obviate the drawbacks of the prior art, a concise but sufficient description of the invention and its usefulness, drawings, and details of the best method of its working.

There are two types of patent documents usually known as patent specification

  1. Provisional specification and
  2. Complete specification.

Provisional Specification

  • While an inventor is in the process of finalizing his invention, he may file a specification known as ‘Provisional Specification’ which is not a full and specific description.
  • It contains only a general description of the invention, its field of application, and anticipated results. The provisional specification need not contain the claim(s).
  • A provisional specification is filed to fix the priority date of the patent. The priority date of the claim is the date on which a specification containing the claim is filed.
  • Normally, the priority date is the date of filing the provisional specification provided the claims are based on the matter disclosed in the provisional specification.
  • During the period between the filing of the provisional specification and the complete specification, the inventor may conduct further research on the subject matter of his invention.
  • The provisional specification is a permanent and independent scientific cum legal document and no amendment is allowed in this.
  • It is not necessary to apply with provisional specification before the complete specification.

Complete Specification

  • The complete specification is the full description of the invention containing all the claims over which the applicant seeks monopoly rights.
  • The object of this specification is to define clearly and with precision the monopoly claimed so that others may know the exact boundaries of the monopoly right of the applicant.
  • Complete specification must be submitted within 12 months of filing the provisional specification.
  • The period can be extended by 3 months. Submission of complete specifications is necessary to obtain a patent.

The contents of a complete specification are:

  1. Title of the invention.
  2. Field to which the invention belongs.
  3. Background of the invention including prior art giving drawbacks of the known inventions and practices.
  4. Complete description of the invention along with experimental results.
  5. Drawings etc. essential for understanding invention.
  6. Claims, which are statements, related to the invention on which legal proprietorship is being sought. Therefore the claims have to be drafted very carefully.

Question 9. What is the Patent Application? Briefly discuss the parts of the patent application.
Answer:

A patent application memorializes the agreement between the inventor and the government office that results in the issuance of a patent. The patent application is in many ways like a contract.

Writing a high-quality patent application is important because it sets out the terms by which the patent owner and others will be bound.

Thus, patent applications should be drafted while keeping important audiences in mind.

The parts of the patent application are as follows:-

1. Patent Application Claims:

  • It is very important to prepare the claims for the invention. The patent agent may even want to sketch out the claims in the disclosure meeting with the inventor. This will often confirm to the patent agent that he has understood the invention.
  • The patent agent may wish to use some sort of “picture claim” in the initial meeting with the inventor since inventors are often unfamiliar with patent claim language.
  • The majority of patent agents prepare several draft patent claims as their first step in writing a patent application. The claims are the legally operative part of a patent application; everything revolves around the claims.
  • Because of the critical importance of claims, the patent agent should carefully revisit them after drafting the specification.
  • Once the claims are completed the patent agent needs to check the drawings and specifications to verify that the claim terms have been appropriately described and disclosed.

Patent Application Detailed Description Or Specification:

  • The detailed description Section, sometimes known as the “preferred embodiment of invention” Section or the “disclosed embodiment of the invention”
  • The section breathes life into the claims and provides a sufficient explanation of the invention for an ordinary person skilled in the art to make and understand the invention.
  • The patent agent cannot amend his application to include new technical disclosure during prosecution.

A patent agent should take care that the patent application must:

  • Reflect the disclosure material provided by the inventors;
  • Provides sufficient information to enable an ordinary artisan to reproduce the invention;
  • Provides sufficient depth so that the claims can be narrowed during patent prosecution to avoid closing prior art.
  • The patent agent need not include in the patent application well-known material that would be needed to make a product associated with the invention.
  • The patent agent must be very careful in his use of language in a patent application.

3. Patent Application Drawings:

  1. The patent agent must prepare good visual supporting materials that describe the invention. Many patent agents would argue that the drawings are the most important part of the patent application after the claims. Some patent laws require that every claimed element be shown in a drawing.
  2. Where possible, the drawings should explain the invention in sufficient detail that reading the detailed section merely confirms in words the information provided in the drawings. In preparing the drawings the patent agent should think of the story he wants to tell and how he wants to tell it.
  3. The elements shown in a patent’s drawings are typically accompanied by a short description in words and a reference number such as “Clock 102″.
  4. The drawing Section should begin with a statement indicating that the drawings are illustrative of one or more embodiments of the invention.

4. Patent Application Background:

  1. The use of a background section varies among the world’s patent regimes.
  2. In some patent systems, the background section serves to disclose to the public the closest prior art applied against the patent application during the examination.
  3. The background Section is typically considered prior art disclosed by the inventor.
  4. Consequently, if the applicant’s inventive disclosure ends up in the background section, the patent examiner may cite this section in the rejection of the applicant’s claims.
  5. A good background section should be fairly short and merely set the stage for the technical disclosure to be provided in the detailed description section.
  6. The background section could describe the prior art at a very high level.
  7. The background section may conclude with a short, crisp statement about the shortcomings of the prior art but this must be written in a manner that does not disclose the solution to be described later in the application.

5. Patent Application Abstract:

  1. The patent abstract should describe the invention very clearly in the fewest possible words.
  2. The patent agent could use a version of the first paragraph of the summary of the invention section as the abstract.

6. Patent Application Summary:

  • As pointed out earlier, not all jurisdictions require a summary of the invention section.
  • The patent agent may find himself reviewing summary sections drafted by foreign patent agents working on his client’s foreign counterpart’s patent application.
  • The patent agent should understand the precise requirements and customary practice regarding a summary of the invention sections in the jurisdictions of interest to his clients.
  • Some patent agents prepare the summary of the invention section by taking each of the independent claims in the patent application and turning them into paragraphs.
  • The advantage of this approach is that the precise words used in the claims will be guaranteed to be in the specification.
  • Many patent agents simply draft the summary of the invention section in a manner that highlights the important aspects of the invention using words drawn from the application’s claims.
  • The summary of the invention section should be one of the last parts of the patent application that the patent agent writes.
  • In preparing the summary of the invention sections, avoid providing some sort of “big picture” summary that goes beyond the claims in any manner.

Patent Information System Question and Answers

Patent Databases And Patent Information System Descriptive Questions

Question 1. A patent search is a search conducted in the patent database to check whether any invention similar to the invention in respect of which the patent is obtained, already exists. Discuss the patent search, patent database, and various types of searches used in patent documentation.
Answer:

The object of Patent search is to evaluate the subject matter invention in comparison to the prior Art.

Prior art refers to scientific and technical information that exists before the effective date of a given patent application.

Prior art may be found in any public documents such as patents, technical publications, conference papers, marketing brochures, products, devices, equipment, processes, and materials.

A prior art search refers to an organized review of prior art contained in public documents, prior art searches can be of various kinds: patentability searches conducted by an inventor before filing a patent application.

Searches are conducted using different kinds of databases, from public databases of issued patents on the internet to exhaustive databases including technical literature. Searches can be done by legal professionals, by scientists, or by researchers, sometimes, defendants in patent litigation even offer bounties for invalidating prior art.

A patentability search may be conducted before the filing of a patent application to gauge the prospects of obtaining broad claim coverage.

The purpose of conducting such a search is to find references related to the claimed invention to assess its patentability.

Searches are typically fast and inexpensive since the patent agent’s clients often do not want to pay for an expensive, thorough search.

Also, it is often presumed that the inventor himself will have a good sense of novelty based on his reading of the literature in his field and by communication with his peers.

Searches are a good way to get information on developments in the field of invention. Prior art searches may sometimes reveal what competitors consider worth protecting. Search results may be a critical factor in deciding whether to file a patent application.

If a prior art search reveals references that anticipate the claimed invention, the inventor and the patent agent should consider how they can “avoid the prior art” by drafting the claims to overcome.

In some cases, a prior art search may reveal problematic patent references. Just because you see a reference that seems similar to the invention does not mean the proposed application should be abandoned.

Patent Databases And Patent Information System

Question 2. Explain the functions of The Patent Information System (PIS) and the National Institute for Intellectual Property Management (NIIPM).
Answer:

The functions of The Patent Information System (PIS) and the National Institute for Intellectual Property Management (NIIPM) are as follows:

PIS

  1. To obtain and maintain a comprehensive collection of patent specification and patent-related literature on a worldwide basis;
  2. To provide technological information in patent or patent-related literature through publication services, search services, and patent copy supply services;
  3. To meet statutory obligation regarding novelty search under the Patent Act, 1970.

Patent-related literature on a worldwide basis to meet the need for technological information of various users in:

  1. R&D establishments
  2. Government Organizations
  3. Industries
  4. Business, and
  5. Inventors.

Other users enable them to take information on business decisions

NIIPM

  1. It is a national center of excellence for training, management, research, and education in the field of Intellectual Property Rights related issues
  2. Caters to the training of Examiners of Patents and Designs
  3. Examiners of Trademarks & Geographical Indications
  4. IP professionals
  5. IP Managers in the Country
  6. Imparting basic education to the user community
  7. Government functionaries and stakeholders involved in the creation
  8. Commercialization and management of intellectual property rights
  9. It also facilitates research on IP-related issues including preparation of study reports and policy analysis of relevance to the Government.

Question 3. List out the individual types of searches in patent documentation.
Answer:

The various types of searches are:

1. Pre-Application Searches (PAS)

Years ago, an invention was just an idea. The patent application process is difficult, time-consuming, and expensive. Therefore, the inventor should conduct a “Pre- Application Searches” (PAS) before filling a patent application. In this search, the inventor should look for:

  1. Printed publications
  2. Public knowledge or
  3. Patent already issued in his country or a foreign country that may relate to the particular invention.

2. State-of-the-Art Searches

It is also known as an “Informative Search”. This search is made to determine the general state-of-the-art for the solution of a given technical problem as background information for R&D activities and to know what patent publications already exist in the field of the technology or research.

This kind of search is especially useful for technology development or technology transfer purposes.

3. Novelty Searches

  1. The objective of this search is to determine the novelty or lack of novelty of the invention claimed in a patent application or a patent already granted, or of an invention for which no application has yet been filed.
  2. This search aims to discover relevant prior art. The basic inventive ideas are formulated in such an unspecified way that many publications will apply to this broad description.
  3. Depending on the outcome of the novelty search, the next decision will be whether to stop or to go ahead with developing the invention.
  4. If nothing of relevance was found, it is easy and you should go ahead. The decision becomes difficult if one or more pertinent documents have been found.
  5. But important is to restrict the search to the appropriate area.
  6. This may be done by identifying a proper place or place for the subject of the searches in the IPC.

4. Patentability or Validity Searches

  1. It is made to locate documents relevant to the determination not only of novelty but also of other criteria of patentability.
  2. This type of search should cover all the technical fields, which may contain material pertinent to the invention.

5. Name Searches

These are searches for locating information about published patent documents involving specific companies or individuals, such as applicants, assignees, patentees, or inventors

6. Technological Activity Searches

  • These searches are for identifying companies and/or inventors who are active in a specific field of technology.
  • These searches are also suitable for identifying countries in which a certain technology is being patented, to know where to turn for obtaining particular information in a given field of technology.

7. Infringement Searches

  • The objective of this search is to locate patent and published patent applications, which might be infringed on by a given industrial activity.
  • This search aims to determine whether an existing patent gives exclusive rights covering that industrial activity or any part of it.

8. Legal Status Searches

A search for this type of investigation is made to obtain information on the validity (status) of a patent or a published patent application, on a given date, under the applicable patent legislation in one or more countries.

Such information can assist in making decisions. For example:

  • Exporting; Or
  • Negotiation Of License Agreement.

It can also give guidance on the value attached to a particular patent by the patentee.

Question 4. Why is a patent search done? What are the various online databases available that provide access to patent documents while conducting a patent search?
Answer:

A patent search is an important step before filing a patent application.

  • A patent search is a search conducted in patent databases as well as in the literature available, to check whether any invention similar to the invention in respect of which patent is to be obtained already exists.
  • Therefore, instead of going forth with the filing, if one conducts the patentability search, one can get a clear idea about the patentability of the invention whether the application should be filed, and the strengths and weaknesses of his invention.
  • Patent information is made available to the public through a variety of databases. Each database covers a particular set of patent documents.
  • Thus, it may be necessary to consult multiple databases to find and then access patent documents relevant to your interests. Internet-based databases are online databases.
  • Anyone who has access to the Internet may be able to browse the full text of published patent documents via free-of-charge databases or commercial databases.
  • Many national and regional patent offices provide free online access to their patent collections as well as to select patent documents from other offices.

For example: The Full-Tent and Full Page Image Database of the United States Patent and Trademark Office (USPTO) is one of the earliest and free online patent information systems.

  • International Patent Classification (IPC) is a hierarchical classification system used primarily to classify and search patent documents according to the technical fields they pertain to.
  • IBM Intellectual Property Network, Intellectual Property Network (IPN) is a free IBM patent site provided by IBM. The database contains:

United States Patents (US):1 971 -present & updated weekly (full tent/ full image)

European Patents – Applications (EP-A): 1979-present, updated weekly (Front page & claims/ full image)

European Patents – Issued (EP-B): 1980- present, updated weekly (front page & claims/ full image)

Patent Abstracts of Japan (JP): 1976- present, updated weekly (front page & claims/ Representative image).

Indian Patent Law Question And Answers

Indian Patent Law

Question 1. Concerning the relevant legal enactments, write short notes on the following: Company Secretary as patent agent.
Answer:

A person shall be qualified as a patent agent if the person is 

  1. Citizen of India
  2. Completed the age of 21 years
  3. Has obtained a degree in science, engineering, or technology
  4. Passed the prescribed qualification exams or worked for not less than ten years as an examiner or discharged the functions of the controller.

In light of the above legal provisions, if a Company Secretary fulfills the above conditions, and satisfies the condition then he will be eligible to act as a patent agent.

Question 2. Concerning the relevant legal enactments, write short notes on the following: Element of ‘novelty’ in an invention
Answer:
One of the prerequisites for anything to be regarded as an invention is something having inventive steps. Novelty means anything new or strange which is thus an indispensable part of anything to be construed as.

Question 3. Regarding the relevant legal enactments, write short notes of the following: Contents of a complete specification
Answer:
The specification is the description of the invention. The specification can be provisional or complete.

The contents of the complete specifications are

  1. Describe the invention, its operation, its use, and its methods.
  2. It must disclose the best method of performing the invention.
  3. Ends with a claim defining the scope of the invention.
  4. Must provide technical information on the invention.
  5. A declaration as to the inventorship of the invention.

Patents Act in India

Question 4. About the relevant legal enactments, write short notes on the following:
Term of patents
Answer:

  1. Provisions relating to the term period of patents are covered under section 53 of the Patents Act 1970.
  2. The term period of patents is 20 years from the date of applying for the patent.
  3. However, non-payment of renewal fees may result in the cessation of the patent.

Question 5. About the relevant legal enactments, write short notes on the following: Restoration of lapsed patents
Answer: Sections 60 to 62 deal with provisions relating to the restoration of lapsed patent, it provides that where the patent has ceased to have effect by reasons of failure to pay any renewal fees within the period prescribed then the patentee may within eighteen months from the date on which the patent ceased to have effect, make an application for restoration of the patent.

If the controller is satisfied that the failure to pay fees was unintentional he may advertise the application. Any person interested may oppose the restoration and after considering the opposition, the controller will make appropriate orders.

Question 6. Concerning the relevant legal enactments, write short notes on the following: Patent Agent
Answer:

A person shall be qualified as a patent agent if the person is

  • Citizen of India
  • Completed the age of 21 years
  • Has obtained a degree in science, engineering, or technology
  • Passed the prescribed qualification exams or worked for not less than ten years as an examiner or discharged the functions of the controller.
  • In the light of the above legal provisions, if a Company Secretary who fulfils the above conditions, satisfies the condition then he will be eligible to act as a patent agent.

Question 6. Regarding the relevant legal enactments, write short notes on the following:
Inventive step, General principles apply to the working of the patented invention.
Answer:

Inventive step

  • The inventive step has been defined in Section 2(1) of the Patents Act, of 1970.
  • Inventive step means a feature of an invention, that involves technical advance as compared to the knowledge or having economic significance or both that makes the invention which is not obvious to a person skilled in the art.

General principles applicable to working of patented invention Some of the principles applicable to working of patented invention covered in Section 83 of the Patents Act, 1970 are as under:

  • Patents are granted to encourage inventions.
  • Patents are not granted to merely enable patentees to enjoy a monopoly.
  • Patents granted do not prohibit the Central Government from taking measures to protect public health.
  • The benefits of patented invention should be available at reasonable prices to the public.
  • Patents granted should act as an instrument to promote public interest especially in sectors of vital importance.
  • Patent rights should not be abused by the patentee.

Indian Patent Law Distinguish Between

Question 1. Distinguish between the following: ‘Invention’ and ‘patentable invention’ under the Patents Act, 1970.
Answer: Section 2 of the Patent Act defines an invention as a new product or process involving an inventive step and capable of Industrial application.

The term ‘inventive step’ has been defined under the Patent Act as a feature of an invention that involves technical advance as compared to the existing knowledge or having economic significance or both that makes the invention not obvious to a person skilled in the art.

Any invention or technology which has not been anticipated by publication in any document or used in the country or elsewhere in the world before the date of filing of a patent application with complete specification, i.e. the subject matter has not fallen into the public domain or that it does not form part of the state of the art, which is registered with Controller of Patent is a patentable invention. Section 3 enumerated the list of inventions which are not patentable.

Question 2. Distinguish between the following: ‘Provisional specification’ and ‘complete specification’ under the Patents Act, 1970.)
Answer:

A provisional specification is a document, which contains the description regarding the nature of an invention.

The description, however, does not contain the details regarding the invention. Also, it does not contain the claims. The provisional specification is filed to claim the priority date of an invention.

The advantage of a provisional specification is that it can be filed as soon as the patent is conceived and for the recorded priority date.

However, the application is only examined after the complete specification has been filed. Section 9 of the Patents Act, 1970 stipulates that where an application for a patent (not being a convention application or an application filed under PCT designation India) is accompanied by a provisional specification, a complete specification shall be filed within twelve months from the date of filing of the application, and if the complete specification is not so filed, the application shall be deemed to be abandoned.

A complete specification is a document, which contains a detailed description of the invention along with the drawings and claims.

Also, the description regarding prior art is included in the complete specification. Section 10 dealing with contents of Specifications provides that every specification, whether provisional or complete, shall describe the invention and begin with a title sufficiently indicating the subject matter to which the invention relates.

  1. Every complete specification is required to
  2. Fully and particularly describe the invention and the operation or use and the method by which it is to be performed; disclose the best method of performing the invention that is known to the applicant and for which he is entitled to claim protection;
  3. End with a claim or claims defining the scope of the invention for which protection is claimed; and
  4. Be accompanied by an abstract to provide technical information on the invention.

Question 3. Distinguish between the following: ‘Patent’ and ‘patent of addition’.
Answer:

  • Patent means a patent granted under the Patent Act, of 1970.
  • A patent is a grant from the government that confers on the grantee, for a limited period, the exclusive privileges of making, selling, and using the invention for which the patent has been granted and also of authorizing others to do so.
  • The patent gives its owner a monopoly right to make & sell the subject matter of the patent for a defined period.

Patent of addition.

  • Patent of Addition is a patent for improvement in, or modification of an invention for which invention, a patent has already been applied for or granted.
  • A patent of addition will remain in force as long as the patent for the original invention remains in force.

Question 4. Distinguish between the following: ‘Surrender of a patent’ and ‘revocation of a patent’ under the Patents Act, 1970.
Answer:

Surrender of patents:

  • Surrender of patents has been provided in Section 63 of the Patents Act, 1970.
  • As per the provisions of the Act, patentee of the patentee can offer to surrender his patent, at any time by giving notice to the controller.
  • The Controller shall publish the offer in the prescribed manner, once the offer is made by the patentee.
  • Any person within the prescribed period after such publication may give notice of opposition to the controller.
  • If the controller is satisfied after hearing the patentee & any opponent, he may accept the offer & by order revoke the patent.

Revocation of Patents:

  • ‘Revocation of patents’ has been covered in Section 64 of the Patents Act, 1970.
  • As per the provisions of the Act, a patent may be revoked by any person, Central Government.
  • If the Central Government is satisfied that a patent is for an invention relating to atomic energy for which no patent can be granted, it may direct the controller to revoke the patent.

Question 5. Mention any five inventions which are not patentable under the Patents Act, of 1970.
Answer:

Inventions not patentable under The Patents Act, 1970 Section 3 of the Patents Act, 1970 provides that the following are not inventions and hence not patentable:

  1. An invention that is frivolous or which claims anything contrary to well-established natural laws
  2. An invention the primary or intended use or commercial exploitation of which could be contrary to public order or morality or which causes serious prejudice to human, animal, or plant life or health or the environment;
  3. The mere discovery of a new form of a known substance that does not result in the enhancement of the known efficacy of that substance the mere discovery of any property or mere new use for a known substance or of the mere use of a known process, machine or apparatus unless such known process results in a new product or employs at least one new reactant;
  4. A substance obtained by a mere admixture resulting only in the aggregation of the properties of the components thereof or a process for producing such substance;
  5. The mere arrangement or re-arrangement or duplication of known devices each functioning independently of one another in a known way;
  6. A method of agriculture or horticulture; any process for the medicinal, surgical, curative, prophylactic diagnostic, therapeutic, or other treatment of human beings or any process for a similar treatment of animals to render them free of disease or to increase their economic value or that of their products;
  7. Plants and animals in whole or any part thereof other than microorganisms but including seeds, varieties, and species and essentially biological processes for production or propagation of plants and animals;
  8. A computer program per se other than its technical application to industry or a combination with hardware;
  9. A literary, dramatic, musical, or artistic work or any other aesthetic creation whatsoever including cinematographic works and television productions;
  10. A mere scheme or rule or method of performing a mental act or method of playing a game;
  11. A presentation of information;
  12. Topography of integrated circuits;
  13. An invention which in effect, is traditional knowledge or which is an aggregation or duplication of known properties of traditionally known component or components.
  14. Section 4 prohibits the grant of a patent in respect of an invention relating to atomic energy falling within Sub-section (1) of Section 20 of the Atomic Energy Act, 1962.

Question 6. Mention five important grounds for opposition to the grant of patents.
Answer:

Section 25 provides that any person can apply for the opposition grant of a patent on the following grounds:

  1. That the applicant had wrongfully obtained the whole or part of the invention from him or the person from whom he claims.
  2. That the invention has been published before the priority date of the claim.
  3. That the invention claimed is publicly known or used
  4. That the invention claimed does not involve any inventive step.
  5. That the invention is not patentable under the Act.
  6. The complete specification does not describe the invention properly.
  7. The complete specification is provided after the specified time.

Question 7. Client XYZ has approached you to get a patent on a drug for curing insomnia. What are the steps involved in registering the patent? Describe with examples.
Answer:

  1. Prior art on the subject must be examined to find out the “novelty”. Patent searches have to be organized.
  2. Similarly, the innovative part has to be seen along with usefulness.
  3. Disclosures and publication of the discovery must be in place.
  4. Claims must be prepared along with detailed specifications, drawings, and manufacturing processes.
  5. The drug must also be physically available to establish its identity.
  6. The patent office will examine the claims and publish the results for objections.
  7. Finally, the patent will be granted and duly registered.

Question 3. ‘There are two kinds of opposition in a grant of patent.’ Discuss the provisions of pre-grant opposition and procedure followed in such situations as per the Patent legislation.
Answer:

There are two types of opposition to the grant of a patent as per the Patent Amendment Act, of 2005. They are:

  1. Pre-Grant Opposition
  2. Post-Grant Opposition.
  3. Pre-Grant Opposition

Section 25 of the Act deals with opposition to the grant of a patent and provides that where an application for a patent has been published but a patent has not been granted, any person may, in writing, represent by way of opposition to the Controller against the grant of a patent on the following grounds and the Controller on request of such person shall hear him and dispose of the representation in the prescribed manner and specified time.

The grounds for the Pre-Grant opposition are as follows:

  • That the applicant for the patent or the person under or through whom he claims wrongfully obtained the invention or any part thereof from him or from a person under or through whom he claims;
  • That the invention so far as claimed in any claim of the complete specification has been published before the priority date of the claim –
  • In any specification filed in pursuance of an application for a patent made in India on or after the 1st day of January 1912;

In India or elsewhere, in any other document: Provided that the ground specified in sub-clause

  • Shall not be available where such publication does not constitute an anticipation of the invention under sub-section (2) or sub-section (3) of section 29;
  • That the invention so far as claimed in any claim of the complete specification is claimed in a claim of a complete specification published on or after the priority date of the applicant’s claim and filed in pursuance of an application for a patent in India, being a claim of which the priority date is earlier than that of the applicant’s claim;
  • That the invention so far as claimed in any claim of the complete specification was publicly known or publicly used in India before the priority date of that claim.

Explanation: For this clause, an invention relating to a process for which a patent is claimed shall be deemed to have been publicly known or publicly used in India before the priority date of the claim if a product made by that process had already been imported into India before that date except where such importation has been for reasonable trial or experiment only;

  • That the invention so far as claimed in any claim of the complete specification is obvious and does not involve any inventive step, having regard to the matter published as mentioned in the clause or having regard to what was used in India before the priority date of the applicant’s claim; That the subject of any claim of the complete specification is not an invention within the meaning of this Act, or is not patentable under this Act;
  • That the complete specification does not sufficiently and clearly describe the invention or the method by which it is to be performed;
  • That the applicant has failed to disclose to the Controller the information required by section 8 or has furnished the information which in any material particular was false to his knowledge;
  • In the case of convention application, the application was not made within twelve months from the date of the first application for protection for the invention made in a convention country by the applicant or a person from whom he derives title;
  • That the complete specification does not disclose or wrongly mention the source or geographical origin of biological material used for the invention;
  • That the invention so far as claimed in any claim of the complete specification is anticipated having regard to the knowledge, oral or otherwise, available within any local or indigenous community in India or elsewhere.
  • The grounds for both pre and post-grant opposition are identical and there is nothing to preclude a pre-grant opponent from subsequently filing a post-grant opposition. However, despite the similarities, there are also several procedural differences between the two types of opposition.
  • The primary difference between pre-grant and post-grant opposition is that though pre-grant proceedings may be initiated by “any person”, only a “person interested” can institute a post-grant opposition.
  • The Indian Patents Act defines a “person interested” as including a person engaged in, or in promoting, research in the same field as that to which the invention relates.
  • Moreover, the Indian Patents Act does not explicitly allow the opponent to be heard in a pre-grant opposition. The opponent has to request a hearing and the rules do not detail how a hearing is to be conducted.
  • The opponent’s right to be heard solely depends upon the discretion of the Controller, who decides the same based on the merit of the opposition.
  • Additionally, the rules are also not clear whether the opponent will be heard in the presence of the applicant.

Opposition Proceedings

Rule 55 dealing with opposition by representation against the grant of a patent requires the representation for opposition under section 25(1) to be filed at the appropriate office within a period not exceeding three months from the date of publication of the application under section 11 A of the Act, or before the grant of a patent, whichever is later and includes a statement and evidence, if any, in support of the representation and a request for hearing if so desired.

The Controller has been empowered to consider such representation only when a request for examination of the application has been filed.

On consideration of the representation, if the Controller thinks that the patent application shall be refused or the complete specification requires amendment, he shall give a notice to the applicant to that effect.

On receiving the notice the applicant shall, if he so desires, file his statement and evidence, if any in support of his application within one month from the date of the notice.

On consideration of the statement and evidence filed by the applicant, the Controller may either refuse to grant a patent on the application or require the complete specification to be amended to his satisfaction before the patent is granted.

After considering the representation and submission made during the hearing if so requested, the Controller shall proceed further simultaneously either rejecting the representation and granting the patent or accepting the representation and refusing the grant of a patent on that application, ordinarily within one month from the completion of the above proceedings.

In the case of Novartis Ag v. Natco Pharma Ltd. on 25 January 2006, a patent application was filed in India on 17th July 1998 by Novartis AG, Switzerland, claiming Switzerland’s priority date of 18th July 1997.

Upon publication, the grant of the patent was opposed by Natco Pharma Ltd., India on 26th May 2005.

The grounds for opposition were:

  1. Anticipation by prior publication
  2. Lack of inventive step
  3. Non-patentability under section 3(d)
  4. Wrongfully claiming the priority

The title compound was already known in a US patent (filed in 1993). The US patent claimed a pharmaceutically acceptable salt of the base compound.

Another Document, “Nature Medicine” (5th May, 1 996) also described the title compound. Also, the claimed salt inherently existed in the most stable form of the salt.

Hence, the claims of the application for the product and process in respect of the title compound stood anticipated by prior publications.

Additionally, based on section 3(d) the product claim amounted to a mere discovery of a new form of the known substance.

Further, the application had claimed Swiss priority, but Switzerland was not a convention country on the date of filing in Switzerland.

Hence, no priority of the Swiss application could be claimed in respect of the Indian application.

Given the above findings and arguments, the Controller ruled that the above patent application cannot proceed for grant of patent.

Question 4. To qualify for the grant of a patent under the Patents Act, of 1970, the invention must be non-obvious. However, determining whether an invention is non-obvious is one of the most difficult tasks in patent law. Discuss.
Answer:

To grant a Patent to an invention, determining whether an invention is non-obvious or not, i.e. determining whether it involves an inventive step or not, is one of the most difficult tasks in Patent law.

The mere coalition of two or more things without the exercise of inventive ingenuity is not a subject matter for a patent. It is an invention if the process or manufacture of an article requires some ingenuity or has an inventive step.

However, simplicity is not necessarily an objection to securing a patent for an invention.

Even if the product and/or the process may be perfectly simple and very common, yet there may be an invention, if the inventor has developed a variant, which will render it more useful.

After all, a disposable razor, a safety pin, and a retractable tape measure all seem obvious now, yet none of these items were obvious at the time they were invented.

The point of view from which it needs to be ascertained as to whether a particular invention is ‘non-obvious or not is that of a person skilled in the particular art to which the invention relates.

An invention that has an inventive step, however small, but having regard to the condition of the state-of-the-art in the particular field on the date of filing or the priority date of the patent application, whichever is earlier constitutes a step forward and may be a subject matter for a patent. Thus, one must review the scope and content of the prior art existing in the particular field to determine if the invention is non-obvious.

Secondly, consideration must be given to the differences or enhancements carried out between the Prior Art and the Invention at hand. Thirdly, as already stated, if the invention would be obvious to a person having primary skill in the art to which the invention pertains, it cannot be patented. The Supreme Court in the case of Bishwanath Prasad Radhey Shyam v. Hindustan Metal Industries interpreted the term inventive step.

In this case, the Court opined that ‘obviousness’ has to be strictly and objectively judged.

The test laid down by the Court to determine obviousness was: ‘Had the document been placed in the hands of a competent craftsman (or engineer as distinguished from a mere artisan), endowed with the common general knowledge at the ‘priority date’, who was faced with the problem solved by the patentee but without knowledge of the patented invention, would he have come up with the invention in question’ Simply put, according to the test, what has to be determined is whether, at the time the invention was made, any other person skilled in the same field as the invention could have come up with the same invention, if faced with the same problem.

Question 4. Many life-saving drugs are patented by multinational pharmaceutical companies and many times there is a shortage of these medicines in the market. In this context explain the working of patented inventions and can government issue licenses for the protection of the general public.
Answer: Section 83 of the Patents Act, 1970 deals with general principles applicable to the working of Patented Invention, it provides that in exercising the powers conferred for working of Patents and Compulsory Licenses, regard shall be had to the following general considerations, namely:

Patents are granted to encourage inventions and to secure that the inventions are worked in India on a commercial scale and to the fullest extent that is reasonably practicable without undue delay;

  • That they are not granted merely to enable patentees to enjoy a monopoly for the importation of the patented article;
  • That the protection and enforcement of patent rights contribute to the promotion of technological innovation and the transfer and dissemination of technology, to the mutual advantage of producers and users of technological knowledge and in a manner conducive to social and economic welfare, and to a balance of rights and obligations;
  • That patents granted do not impede the protection of public health and nutrition and should act as instruments to promote public interest, especially in sectors of vital importance for the socio-economic and technological development of India;
  • That patents granted do not in any way prohibit the Central Government from taking measures to protect public health;
  • That the patent right is not abused by the patentee or person deriving title or interest on a patent from the patentee, and the patentee or a person deriving title or interest on a patent from the patentee does not resort to practices which unreasonably restrain trade or adversely affect the international transfer of technology; and That patents are granted to make the benefit of the patented invention available at reasonably affordable prices to the public.

Compulsory Licenses

The provisions for Compulsory Licenses are made in the Patents Act, of 1970 to prevent the abuse of the grant of a Patent as a monopoly and to make the way for commercial exploitation of the invention by any interested person.

According to Section 84 of the Patents Act, 1970, any person interested can make an application for grant of compulsory license for a Patent after three years from the date of grant of that Patent on any of the following grounds:

  1. That the reasonable requirements of the public for the patented invention have not been satisfied, or
  2. That the patented invention is not available to the public at a reasonably affordable price, or
  3. That the patented invention has not worked in the territory of India.

An application for Compulsory License may be made by any person notwithstanding that he is already the holder of a license under the Patent and no person shall be estopped from alleging that the reasonable requirements of the public to the patented invention are not satisfied or that the patented invention is not worked in the territory of India or that the patented invention is not available to the public at a reasonably affordable price because of any admission made by him, whether in such a license or otherwise or because of his having accepted such a license.

Thus, while the law confers some exclusive monopoly rights upon the individuals (Patent Holders) in respect of their inventions as a credit or reward for the time, efforts, and capital spent by them in coming up with such an invention, it equally draws a balance by making provisions thereby ensuring that grant of such a right does not run counter to the interest of society / general public and thus in case there is a shortage of any life-saving drug which is a subject matter of Patent by a Multinational Corporation, the Government is entitled to direct such Patent Holder or itself grant the right to a potential player to carry on production of such life-saving product to ensure that there is no short supply of it in the public.

Question 5. Ajit developed a method of hedging against price fluctuation in the energy market. He used a simple concept of mathematics and a familiar statistical approach to achieve the above purpose. Ajit termed it as an innovative procedure to calculate the risk at this competitive time. Advise him on the patentability of his innovation
Answer:

As per the Patents Act, of 1970, the condition of patentability has to be satisfied. The criteria are threefold, i.e., Novelty, Inventive Step, and Industrial Applicability. However, section 3 of the Act lists the inventions as nonpatentable subject matter.

Section 3 (k) provides those mathematical or business methods or a computer program perse or algorithms are not inventions and therefore not patentable. Mathematical methods are considered to be the act of mental skill.

A method of calculation, formulation of equations, finding square roots, cube roots, and all other methods directly involving mathematical methods are therefore not patentable.

With development in computer technology, mathematical methods are used for writing algorithms and computer programs for different applications and the claimed invention is sometimes camouflaged as one relating to the technological development rather than the mathematical method itself.

A mathematical method is carried out on numbers and provides a result in numerical form and is not patentable. If low-quality business methods or mathematical methods are allowed to be patentable, it may damage the public perception of the patent system.

It may create enormous monopoly power much more than the cost of so-called invention. The given problem since it is a simple concept of mathematics and a familiar statistical approach. It is therefore not patentable.

Indian Patent Law Practical Questions

Question 1. Sitca Healthcare (hereinafter referred to as ‘applicant’) filed a divisional patent application on 1 1 m May 2007 numbered 00077/np/2007 for an invention “crystal modification of active pharmaceutical ingredient” for the patent application number 00072/np/2007 having german priority patent application number 1404/97dated 10th june, 2003 It was observed during the substantial examination that claims 1-10 of parent application were reproduced verbatim in the claimed invention as claims 1-10.

If the subject matter of any claimed invention relates to more than one invention, an applicant can file further application any time before the grant of patent u/s 16(1) of the Patents (Amendment) Act, 2005.

Since claims of the claimed invention and claims of the parent application are verbatim, hearing was offered to the applicant to decide the allowability of divisional status to the claimed invention u/s 16 of the Act.

The Applicant replied that the Controller can offer a hearing for deciding the divisional status only after sending a gist of objections to the applicant and allowing him to file a reply.

He further stated that the Controller cannot decide the divisional status unless examined and passed through sections 12,14 and 15 of the Patents Act.

There is no such direction or any other guiding principle laid down in the Act as to how a Controller should proceed with the divisional status of an application.

The Controller observed that the application shall define distinct subject matter when compared with claims of the parent application, which is the first and foremost requirement to qualify as a divisional application u/s 1 6 of the Act.

Once claims of the claimed invention fulfill the requirement for the divisional application u/s 1 6, it can be further allowed for substantial examination, which will save time and effort of the patent administration and also it is a logical approach as well.

Since the subject matter of claims 1-10 is verbatim with claims 1-10 of the parent application, there is no need for conducting a substantial examination to assess the novelty, inventive step, and other patentability criteria for the present application.

The applicant claims the same set of claims submitted in the parent application in multiple further applications.

The Controller further observed that the reason for filing such a divisional application is to prosecute once again the same set of claims already claimed in the abandoned parent application and also to extend the life of the application.

If the subject matter of claims of the further application is not distinct from the claims of the parent application, divisional status shall not be granted.

It is, therefore, a wilful attempt to obtain multiple patents for a single subject matter through many further applications by misusing the provision of section 16 of the Act.

Even though each component of claims 1-10 of the divisional application is verbatim with the claims 1-10 of the parent case, the intention of the applicant by hook or crook is to extend the life of the application utilizing the divisional route, that too, on the last date of submitting reply to first examination report (FER) of the parent case.

There were 10 claims at the time of filing the claimed invention where claims 1-9 related to ‘modification A of the compound 1-(2, 6-difluorobenzyl) – 1H- 1,2,3-trizole-4-carboxamide (namide)’ (hereinafter referred as ‘product claims’) and claim 10 related to pharmaceutical preparation.

On 15 July 2011, the applicant amended the originally filed 10 claims into 12 claims by filing Form-13 u/s 57 and u/s 81(1) of the Act and Rules.

The applicant amended ‘modification A of the compound namide’ in all the claims 1-10 into ‘crystal modification A of the compound namide’ and also added two new claims relating to ‘method of preparing A of the compound namide’ which is neither claimed in the International application nor the National Phase application.

In response to the FER, the applicant further amended claim 11 by incorporating many additional features including identity and quality of the solvent for recrystallization, filtration conditions necessary to generate a suspension with initial crystal, stirring time, and temperature.

The patent application was opposed by way of pre-grant representation u/s 25(1) of the Act, by the Indian Pharmaceutical Alliance, a Society registered under the Societies Registration Act, 1860 (hereinafter referred to as ‘Opponent’).

After the hearing notice was served, the Applicant further amended all the ‘product claims’ into ‘21 method claims as a method for preparing crystal modification A of the compound namide’.

In the earlier amendment, the applicant introduced a new matter in claim 11 & 12 ‘method of preparing A of the compound namide’ but the later amendment converted all the claims into ‘method for preparing crystal modification A of the compound namide’, the principal claim.

The applicant submitted a set consisting of 13 method claims which they desired to rely on during the hearing, (hereinafter referred to as ‘amended claims’).

The patent agent for the applicant submitted that Rule 55 (5) of the Patents Rules, 2003 gives power to the Controller to require the complete specification to be amended to his satisfaction during the opposition, before proceeding to refuse the application.

The patent agent for the applicant submitted that amendment of claims is a primary right conferred upon the applicant by the Patents Act and it is possible to amend the claims to avoid overlapping with the citations, and the Controller should require the same rather than proceeding to reject the application.

It is further submitted that the amended claims on file now fall well within the scope of Section 59.

The patent agent submitted that it is a well-accepted fact that ‘product claims’ cover in its scope the process of its manufacture as well and therefore the amendment made to claims of this application satisfies

Section 59 of the Act is that the amendment is made by way of a ‘disclaimer’ wherein the scope of the product is disclaimed and the claims are now restricted to the process of its preparation.

Also that the scope of the claims is not extended and no new subject matter is added.

The process claims are well supported by the description, especially on page 2 and last 3 lines, page 3 and 2nd para, and in examples 1, 2, 3, 4, and 5.

Keeping the above in view, prepare your arguments for the opponent of the patent, covering the division of the patent, the right of opposition under the Patents Act, and the legal provisions applicable to modification of specification.

Issues in the present case

  1. Can the Applicant Impose, upon the Controller to have and accept the division application?
  2. Does the applicant have the right to amend the patent specification and how?
  3. Can the applicant amend the specification by replacing product claims with process claims in compliance with the provisions of the Patent Act 1970?

The opponents would respectfully like to draw the kind attention of the Learned Controller towards the following findings and facts in the matter of the finally amended claims.

Section 16 of the Patent Act, of 1970 dealt with the power of the Controller to make orders respecting the division of application.

A person who has made an application for a patent under this Act may, at any time before the grant of the patent, if he so desires, remedy the objection raised by the controller on the ground that the claims of the complete specification related to more than one invention, file a further application in respect of an invention disclosed in the provisional or complete specification already filed in respect of the first mentioned application.

Since claims of the claimed invention and claims of the parent application are verbatim.

There is no such direction or any other guiding principle laid down in the Act, as to how the controller should proceed with the divisional status of an application.

Further application shall define distinct subject matter when compared with claims of the parent application, which is the first and foremost requirement to qualify as divisional application u/s 16 of the Act.

Applicant cannot claim the same set of claims claimed in the parent application in multiple further applications.

It is observed that the reason for filling such a divisional application is to prosecute once again the same set of claims. If the subject matter of claims of the further application is not distinct from the claims of the parent application, divisional status shall not be granted.

In the case of LG Electronics Inc. Vs Controller of Patents, the Honble Intellectual Property Appellate Board (IPAB) held that the applicant cannot take undue advantage of Section16 for extending to prosecute the same subject matter.

Section 57 deals with the Amendment of application and specification or any document relating thereto before the Controller.

1. Subject to the provisions of Section 59, the Controller may, upon application made under this section in the prescribed manner by an applicant for a patent or by a patentee, allow the application for the patent or the complete specification or any document relating thereto to be amended subject to such condition,

If any as the Controller thinks fit: Provided that the Controller shall not pass any order allowing or refusing an application to amend an application for a patent or a specification or any document relating thereto under this section, while any suit before a Court the infringement of the patent or any proceeding before the High Court for the revocation of the patent in pending, whether the suit or proceeding commenced before or after the filing of the application to amend.

Types Of Intellectual Property Rights Origin And Development Question And Answers

Types Of Intellectual Property-Origin And Development- An Overview

Question 1. One Unniithan conceived the idea of a reality TV Show SWAYAMWAR concerning matchmaking. He shared the concept note with filmmaker Ramesh Chandra. To his surprise and shock, he came across a newspaper report about Ramesh Chandra planning to come out with a big basket match-making show using his concept. Has there been a violation of an intellectual property right? Can Unnithan institute a suit seeking an injunction? Give reasons and refer to case law, if any.
Answer: The facts given in the question are similar to the case G. Anand vs. Delux Films and Ors. AIR 1978 SC 1613 in which the Appellant R. G. Anand wrote and produced a play called ‘Hum Hindustani’ in 1953 which received huge success and was re-staged numerous times. The appellant narrated the entire play ‘Hum Hindustani’ to the Respondents.

The appellant had elaborate discussions regarding filming the play on January, 1 955. However, no further communication was made to the Appellant post the discussion. However, the respondents proceeded to make the film and the appellant filed a suit claiming copyright infringement.

The Hon’ble Supreme Court held that there exists no copyright to an idea, subject matter, themes, plots, or historical or legendary facts. Infringement is restricted only to the form, manner, arrangement, and expression of the idea by the author of the copyrighted work.

The Court should settle on whether the similarities are substantial or fundamental or not to the mode of expression adopted in the work. If a substantial or fundamental portion has been copied, then it would amount to infringement.

The other reliable test to ascertain whether there is an infringement or not is to analyze the impression created on the spectator or reader after reading or watching the works (Lay Observer Test).

Unlike the case with patents, copyright protects the expressions and not the ideas.

There is no copyright in an idea. In M/s Mishra Bandhu Karyalaya & Others v. Shivaratanlalkoshalair 1970MP261, it has been held that the laws of copyright do not protect ideas, but they deal with the particular expression of ideas.

It is always possible to arrive at the same result from independent sources. The rule appears to be settled that the compiler of a work in which absolute originality is of necessary excluded is entitled, without exposing himself to a charge of piracy, to make use of preceding works upon the subject, where he bestows such mental labor upon what he has taken and subjects it to such revision and correction as to produce an original result.

Mr. Anil Gupta and Anr. v. Mr. Kunal Dasgupta and Ors 13 [97(2002) DLT 257], the Delhi High Court was encountered with a case related to “concept note”.

The Plaintiff alleged that he had narrated a concept of “Swyaamwar” for a reality show in which a bride will traditionally choose her groom in the form of a Swyaamwar. The idea was narrated to the Defendant.

The defendant after a few months launched his TV reality show, “Shubh Vivah” based on the concept of Plaintiff, shared with him in confidence.

Justice Vijender Jain of Hon’ble Delhi High Court relied on the judgment of Terrapin v Builders Supply Co (Hayes) to evoke the doctrine of confidentiality and found force in the argument and thereby restrained the Defendants and granted an injunction in favor of the Plaintiff. Therefore, in the given case, it is simply an idea, that is not protected by the law of copyrights.

A simple idea is not an intellectual property right as stated in the above case laws. Therefore, there is no violation of intellectual property rights and Unnithan Institute cannot file a suit seeking an injunction.

Types Of Intellectual Property Short Question and Answers

Question 1. Write a brief note discussing the relativity between Intellectual Property and Business.
Answer: Intellectual Property vis-a-vis Business: A Rationale of Relativity: In today’s world, the abundant supply of goods and services on the markets has made life very challenging for any business, big or small.

In its ongoing quest to remain ahead of competitors in this environment, every business strives to create new and improved products (goods and services) that will deliver greater value to users and customers than the products offered by competitors.

To differentiate their products – a prerequisite for success in today’s markets – businesses rely on innovations that reduce production costs and/or improve product quality.

In a crowded marketplace, businesses have to make an ongoing effort to communicate the specific value offered by their product through effective marketing that relies on well-thought-out branding strategies.

In the current knowledge-driven, private sector-oriented economic development paradigm, the different types of intangible assets of a business are often more important and valuable than its tangible assets. A key subset of intangible assets is protected by what are labeled collectively as intellectual property rights (IPRs).

These include trade secrets protection, copyright, design and trademark rights, and patents, as well as other types of rights.

IPRs create tradable assets out of products of human intellect and provide a large array of IPR tools on which businesses can rely to help drive their success through innovative business models.

All businesses, especially those that are already successful, nowadays have to rely on the effective use of one or more types of intellectual property (IP) to gain and maintain a substantial competitive edge in the marketplace.

Business leaders and managers, therefore, require a much better understanding of the tools of the IP system to protect and exploit the IP assets they own, or wish to use, for their business models and competitive strategies in domestic and international markets.

Question 2. Write a brief note on the utility model.
Answer: Utility Models: A utility model is an exclusive right granted for an invention, which allows the right holder to prevent others from commercially using the protected invention, without his authorization for a limited period.

In its basic definition, which may vary from one country (where such protection is available) to another, a utility model is similar to a patent.

Utility models are sometimes referred to as “petty patents” or “innovation patents.

Only a small but significant number of countries and regions provide the option of utility model protection. At present, India does not have legislation on Utility models.

The main differences between utility models and patents are the following: The requirements for acquiring a utility model are less stringent than for patents.

While the requirement of “novelty” is always to be met, that of “inventive step” or “non-obviousness” may be much lower or absent altogether.

In practice, protection for utility models is often sought for innovations of a rather incremental character that may not meet the patentability criteria.

The term of protection for utility models is shorter than for patents and varies from country to country (usually between 7 and 10 years without the possibility of extension or renewal).

In most countries where utility model protection is available, patent offices do not examine applications as to substance before registration.

This means that the registration process is often significantly simpler and faster, taking on an average of six months.

Utility models are much cheaper to obtain and to maintain. In countries, utility model protection can only be obtained for certain fields of technology, and only for products but not for processes. Utility models are considered suitable particularly for SMEs that make “minor” improvements to, and adaptations of, existing products.

Utility models are primarily used for mechanical innovations. The “Innovation patent,” launched in Australia some time back was introduced as a result of extensive research into the needs of small and medium-sized enterprises, to provide a “low-cost entry point into the intellectual property system.”

Question 2. Write a brief note on Biodiversity and IPR.
Answer: In simple terms, the diversity among various life forms within the Biosphere refers to biodiversity. Biodiversity is the foundation of life on Earth.

It is crucial for the functioning of ecosystems which provide us with products and services without which we cannot live. By changing biodiversity, we strongly affect human well-being and the well-being of every other living creature.

Biodiversity is normally classified under 3 major categories: ecosystem diversity, representing the principal biogeographic regions and habitats; species diversity, representing variability at the level of families, genera, and species; and genetic diversity, representing the large amount of variability occurring within a species.

Diverse activities and actions have been taken by several stakeholders at local, state, national, and international levels to conserve/protect valuable resources such as biodiversity to draw the benefits accrued in them for society.

The Convention on Biological Diversity (CBD) 1992: Opened for signature at the Earth Summit in Rio de Janeiro in 1992, and entering into force in December 1993, the Convention on Biological Diversity is an international treaty for the conservation of biodiversity, the sustainable use of the components of biodiversity and the equitable sharing of the benefits derived from the use of genetic resources.

The interface between biodiversity and intellectual property is shaped at the international level by several treaties and processes, including at the WIPO, and the TRIPS Council of the WTO. With 193 Parties, the Convention has near universal participation among countries.

The Convention seeks to address all threats to biodiversity and ecosystem services, including threats from climate change, through scientific assessments, the development of tools, incentives, and processes, the transfer of technologies and good practices, and the full and active involvement of relevant stakeholders including indigenous and local communities, youth, NGOs, women, and the business community.

The Cartagena Protocol on Biosafety is a subsidiary agreement to the Convention.

It seeks to protect biological diversity from the potential risks posed by living modified organisms resulting from modern biotechnology.

The treaty defines biodiversity as “the variability among living organisms from all sources Including, inter alia, terrestrial, marine and other aquatic ecosystems and the ecological complexes of which they are part, this includes diversity within species, between species and of ecosystems.

The Convention reaffirms the principle of state sovereignty, which grants states sovereign rights to exploit their resources under their environmental policies together with the responsibility to ensure that activities within their jurisdiction or control do not cause damage to the environment of other states.

The Biodiversity Convention also provides a general legal framework regulating access to biological resources and the sharing of benefits arising from their use. India is a party to the Convention on Biological Diversity (1992).

The Convention on Biological Diversity establishes important principles regarding the protection of biodiversity while recognizing the vast commercial value of the planet’s store of germplasm.

However, the expansion of international trade agreements establishing a global regime of intellectual property rights creates incentives that may destroy biodiversity, while undercutting social and economic development opportunities as well as cultural diversity.

The member countries were pressured to change their IPR laws to conform to the TRIPS agreement.

Types Of Intellectual Property Descriptive Questions

Question 1. What are the points, one should consider while adopting a Trademark?
Answer:

A trademark is a word, phrase, symbol, or design that distinguishes the source of products (trademarks) or services (service marks) of one business from its competitors.

To qualify for patent protection, the mark must be distinctive.

For example, the Nike “swoosh” design identifies athletic footwear made by Nike.

The Trade Marks Act 1999 (“TM Act”) provides, inter alia, for registration of marks, filing of multi-class applications, the renewable term of registration of a trademark as ten years as well as recognition of the concept of well-known marks, etc.

It is pertinent to note that the letter “R” in a circle i.e. with a trademark can only be used after the registration of the trademark under the TM Act.

Question 2. Discuss the process for the enforcement of Patent Rights.
Answer: Enforcement of Patent Rights: It is pertinent to note that the patent infringement proceedings can only be initiated after the grant of a patent in India but may include a claim retrospectively from the date of publication of the application for the grant of the patent.

Infringement of a patent consists of the unauthorized making, importing, using, offering for sale, or selling of any patented invention within India.

Under the (Indian) Patents Act, of 1970 only a civil action can be initiated in a Court of Law.

Like trademarks, the relief that a court may usually grant in a suit for infringement of a patent includes permanent and interim injunction, damages or account of profits, delivery of the infringing goods for destruction, and cost of the legal proceedings.

Question 3. Discuss the salient features of the Design Act, of 2000.
Answer:

The salient features of the Design Act, of 2000 are as under:

  1. Enlarging the scope of definition of the terms “article”, “design” and introduction of definition of “original”.
  2. Amplifying the scope of “prior publication”.
  3. Introduction of provision for delegation of powers of the Controller to other officers and stipulating statutory duties of examiners.
  4. Provision of identification of non-registrable designs.
  5. Provision for substitution of applicant before registration of a design.
  6. Substitution of Indian classification by internationally followed system of classification.
  7. Provision for inclusion of a register to be maintained on the computer as a Register of Designs.
  8. Provision for restoration of lapsed designs.
  9. Provisions for appeal against orders of the Controller before the High Court instead of the Central Government.
  10. Revoking of a period of secrecy of two years of a registered design.
  11. Providing for compulsory registration of any document for transfer of rights in the registered design.
  12. Introduction of additional grounds in cancellation proceedings and provision for initiating the cancellation proceedings before the Controller in place of the High Court.
  13. Enhancement of quantum of penalty imposed for infringement of a registered design.
  14. Provision for grounds of cancellation to be taken as a defense in the infringement proceedings to be in any court not below the Court of District Judge.
  15. Enhancing the initial period of registration from 5 to 1 0 years, to be followed by a further extension of five years.
  16. Provision for allowance of priority to other convention countries and countries belonging to the group of countries or inter-governmental organizations apart from the United Kingdom and other Commonwealth Countries.
  17. Provision for the avoidance of certain restrictive conditions for the control of anti-competitive practices in contractual licenses.

Question 4. Discuss the Salient features of Geographical indications.
Answer:

Geographical Indications: Until recently, Geographical indications were not registrable in India and in the absence of statutory protection, Indian geographical indications had been misused by persons outside India to indicate goods not originating from the named locality in India.

Patenting turmeric, neem, and basmati are the instances that drew a lot of attention to this aspect of Intellectual property.

Mention should be made that under the Agreement on Trade-Related Aspects of Intellectual Property Rights (TRIPS), there is no obligation for other countries to extend reciprocal protection unless a geographical indication is protected in the country of its origin.

India did not have such a specific law governing geographical indications of goods that could adequately protect the interest of producers of such goods.

Role Of International Institutions in Patents Act

Role Of International Institutions Descriptive Questions

Question 1. State the relationship between the TRIPS agreement and the ‘pre-existing international conventions’ covered under it.
Answer:

  1. The TRIPS Agreement says WTO member countries must comply with the substantive obligations of the main conventions of WIPO the Paris Convention on industrial property, and the Berne Convention on copyright (in their most recent versions).
  2. Except for the provisions of the Berne Convention on moral rights, all the substantive provisions of these conventions are incorporated by reference. They therefore become obligations for WTO member countries under the TRIPS Agreement – they have to apply these main provisions and apply them to the individuals and companies of all other WTO members.
  3. The TRIPS Agreement also introduces additional obligations in areas that were not addressed in these conventions or were thought not to be sufficiently addressed in them.
  4. The TRIPS Agreement is therefore sometimes described as a “Berne and Paris- plus” Agreement.
  5. The text of the TRIPS Agreement also makes use of the provisions of some other international agreements on intellectual property rights:
  6. WTO members are required to protect integrated circuit layout designs under the provisions of the Treaty on Intellectual Property in Respect of Integrated Circuits (IPIC Treaty) together with certain additional obligations.
  7. The TRIPS Agreement refers to several provisions of the International Convention for the Protection of Performers, Producers of Phonograms, and Broadcast in Organizations (Rome Convention), without entailing a general requirement to comply with the substantive provisions of that Convention.

Question 2. What is the relationship between the TRIPS Agreement and the Pre-existing International Conventions covered under it?
Answer: The TRIPS Agreement says WTO member countries must comply with the substantive obligations of the main conventions of WIPO, the Paris Conventions on Industrial Property, and the Berne Convention on Copyright (in their most recent versions).

Except for the provisions of the Berne Convention on moral rights, all the substantive provisions of these conventions are incorporated by reference.

They, therefore, become obligations for WTO member countries under the TRIPS Agreement – they have to apply these main provisions and apply them to the individuals and companies of all other WTO members.

The TRIPS Agreement also introduces additional obligations in areas that were not addressed in these conventions or were thought not to be sufficiently addressed in them. The TRIPS Agreement is therefore sometimes described as a Berne and ‘Paris-Plus’ Agreement.

The TRIPS Agreement contains references to the provisions of certain pre-existing intellectual property conventions.

According to Article 2.1 of the Agreement, the WTO Members shall, in respect of Parts 2, 3, and IV of the Agreement, comply with Articles 1 through 12, and Article 19, of the Paris Convention (1967) (the Stockholm Act of 14 July 1967 of the Paris Convention for the Protection of Industrial Property).

Article 9.1 of the Agreement requires Members to comply with Articles 1 through 21 Berne Convention (1 971) and the Appendix thereto (the Paris Act of 24 July,

1971 of the Berne Convention for the Protection of Literary and Artistic Works). However, Members do not have rights or obligations under the TRIPS Agreement in respect of the rights conferred under Article 6b of that Convention, i.e. the moral rights, or of the rights derived therefrom.

As regards the protection of the layout designs of integrated circuits, Article 35 of the Agreement requires Members to comply with Articles 2 through 7 of the Treaty on Intellectual Property in respect of Integrated Circuits, adopted at Washington on 26 May 1989.

The Agreement contains some references to certain provisions of the Rome Convention (the International Convention for the Protection of Performers, Producers of Phonograms and Broadcasting Organizations, adopted at Rome on 26 October 1961).

However, there is no general obligation to comply with the substantive provisions of that Convention.

The TRIPS Agreement provisions on copyright and related rights clarify or add obligations on several points:

The TRIPS Agreement ensures that computer programs will be protected as literary works under the Berne Convention and outlines how databases must be protected under copyright; It also expands international copyright rules to cover rental rights.

Authors of computer programs and producers of sound recordings must have the right to prohibit the commercial rental of their works to the public.

A similar exclusive right applies to films where commercial rental has led to widespread copying, affecting copyright owners’ potential earnings from their films; It says performers must also have the right to prevent unauthorized recording, reproduction, and broadcast of live performances (bootlegging) for no less than 50 years.

Producers of sound recordings must have the right to prevent the unauthorized reproduction of recordings for 50 years.

The test of the TRIPs agreement also makes use of the provisions of some other international agreements on intellectual property rights:

WTO members are required to protect “Integrated Circuit Layout Designs” under the provisions of the Treaty on Intellectual Property In Respect of Integrated Circuits (IPIC Treaty) together with certain additional obligations.

The TRIPS Agreement refers to several provisions of the International Convention for the Protection of Performers, Producers of Phonograms, and Broadcasting Organizations (Rome Convention) without entailing a general requirement to comply with the substantive provisions of that Convention.

Article 2 of the TRIPS Agreement specifies that nothing in Paris 1 to 4 of the agreement shall derogate from existing obligations that members may have to comply with each other under the Paris Convention, the Berne Convention, the Rome Convention, and the Treaty on Intellectual Property in respect of Integrated circuits

Question 3. The Universal Copyright Convention (UCC) was developed by the United Nations Educational, Scientific and Cultural Organization (UNESCO) as an alternative to the Berne Convention for those states that disagreed with aspects of the Berne Convention, but still wished to participate in some form of multilateral copyright protection. Identify the limitation of the Berne Convention and why the Berne Convention states also became a party to the UCC.
Answer:

The first multilateral agreement on copyright was the Berne Convention which was concluded in 1886 and was meant to protect literary and artistic works.

A country joining the Convention has to provide copyright protection to literary and artistic works of member countries in its territory and is also entitled to enjoy reciprocal protection from others. Ninety countries are at present members of the Berne Convention.

The post-Second World War era saw the emergence of the need for protecting copyright on a universal basis. Till then countries in North America were not a party to the Berne Convention and copyright protection in these countries was governed by various national and regional agreements.

The Universal Copyright Convention (UCC), adopted in Geneva, Switzerland, in 1952, is one of the two principal international conventions protecting copyright; the other is the Berne Convention.

The UCC was developed by the United Nations Educational, Scientific and Cultural Organization (UNESCO) as an alternative to the Berne Convention for those states that disagreed with aspects of the Berne Convention but still wished to participate in some form of multilateral copyright protection.

These states included developing countries as well as the United States and most of Latin America.

The former thought that the strong copyright protections granted by the Berne Convention overly benefited Western, developed, copyright-exporting nations, whereas the latter two were already members of the Buenos Aires Convention, a Pan-American copyright convention that was weaker than the Berne Convention.

The Berne Convention states also became a party to the UCC, so that their copyrights would exist in non-Berne convention states.

In 1973, the Soviet Union joined the UCC. The United States only provided copyright protection for a fixed, renewable term, and required that for a work to be copyrighted it must contain a copyright notice and be registered at the Copyright Office.

The Berne Convention, on the other hand, provided for copyright protection for a single term based on the life of the author and did not require registration or the inclusion of a copyright notice for copyright to exist.

Thus the United States would have to make several major modifications to its copyright law to become a party to it.

At the time the United States was unwilling to do so. The UCC thus permits those states which had a system of protection similar to the United States for fixed terms at the time of signature to retain them.

Eventually the United States became willing to participate in the Berne convention, and change its national copyright law as required. In 1989 it became a party to the Berne Convention as a result of the Berne Convention Implementation Act of 1988.

Read the following case on patent law and answer the questions that follow:

Trade-Related Aspects of Intellectual Property Rights (TRIPS) defines geographical indication as “goods originating in the territory of a member, or a region or locality in that territory, where a given quality, reputation, or another characteristic of the goods is essentially attributable to its geographical origin”.

In the Indian legal system, Geographical Indication (Gl) is governed by the Geographical Indications of Goods (Registration and Protection) Act, of 1999. A case relating to GI is that of ‘Basmati rice’ being patented in the United States of America (USA).

Basmati rice is regarded as the ‘queen of fragrance or the perfumed one’ and is also acclaimed as the ‘crown jewel’ of South Asian rice.

It is treasured for its intense fragrance and taste, famous in national as well as international markets. This kind of rice has been grown in the Himalayan hills, Punjab, Haryana, and Uttar Pradesh since times immemorial.

Basmati is the finest quality of rice, long-grained, and the costliest in the world. Agricultural and Processed Food Products Export Development Authority (APEDA) states India to be the second largest exporter of rice after China.

The USA is a major importer of Basmati rice totalling 45,000 tonnes. An important case in the history of Gl and bio-piracy arose in 1997. Royal Rice Tec Inc.

(RRT), a tiny American rice company with an annual income of around US $10 million and working staff totaling 120, produces a small fraction of the world’s (Basmati-like) rice with names ‘Kasmati’ and Texmati’.

RRT had been trying to enter the world rice market for a long but in vain.

On 2nd September 1997, RRT was issued a patent for its Basmati rice lines and grains by the United States Patent and Trademark Office (USPTO) bearing patent number 5663484, which gave it the ultimate rights to call the odoriferous rice ‘Basmati’ within the US and label it the same for export internationally.

According to RRT, its invention of Basmati rice relates to novel rice lines, which affords novel means for determining cooking and which has unique starch properties, etc.

Since times immemorial, the majority of farmers from India have been sustaining the cultivation of Basmati rice and have been among the leading rice producers of the world. The cultivation of rice is not merely a life sustainer but also a part of socio-culture in India. Basmati rice produced in India has been exported to countries like Saudi Arabia and the UK. Basmati is a ‘brand name’ of the rice grown in India.

Two Indian NGOs, namely, the Centre for Food Safety, an international NGO that campaigns against bio-piracy, and the Research Foundation for Science, Technology, and Ecology, an Indian environmental NGO, objected to the patent granted by USPTO and filed petitions in the USA. Council for Scientific and Industrial Research (CSIR), a Government of India organization also objected to the patent granted to RRT.

They demanded an amendment to US Rice Standards because the term ‘Basmati’ can be used only for the rice produced/grown in the territories of India.

According to RRT, the invention relates to novel rice lines and to plants and grains of these lines. The invention also relates to a novel means for determining the cooking and starch properties of rice grains and identifying desirable rice lines.

Specifically, one aspect of the invention relates to novel rice lines whose plants are semi-dwarf in stature and give high-yielding rice grains having characteristics similar or superior to those of good quality Basmati rice. Another aspect of the invention relates to novel rice lines produced from novel rice lines.

The invention provides a method for breeding these novel lines. A third aspect relates to the starch index (SI) of the rice grain, which can predict the grain’s cooking and starch properties and select desirable segregates in rice breeding programs.

The Government of India reacted immediately after learning of the Basmati patent issued to RRT, stating that it would approach the USPTO and urge them to re-examine the patent to a US firm to grow and sell rice under the Basmati brand name, to protect India’s interests, particularly those of growers and exporters.

Furthermore, a high-level Inter-Ministerial Group comprising representatives of the Ministries and Departments of Commerce, Industry, External Affairs, Agriculture and Bio-Technology, CSIR, All India Rice Exporters Association (AIREA), APEDA and Indian Council of Agricultural Research (ICAR) was mobilized to begin an in-depth examination of the case.

In the presence of widespread uprising among farmers and exporters, India as a whole feels confident of being able to successfully challenge the Basmati patent by RRT, which got a patent for three things: Growing rice plants with certain characteristics identical to Basmati, the grain produced by such plants and the method of selecting the rice plant based on a starch index (SI) test devised by RRT.

The lawyers plan to challenge this patent on the basis that the abovementioned plant varieties and grains already exist and thus cannot be patented.

In addition, they accessed some information from the US National Agricultural Statistics Service in its Rice Yearbook 1997, released in January 1998 to the effect that almost 75 percent of US rice imports are Jasmine rice from Thailand and most of the remainder are from India, Varieties that cannot be grown in the US.

This piece of information is sought to be used as a weapon against RRT’s Basmati patent. Indians feel that the USPTO’s decision to grant a patent for the prized Basmati rice violates the International Treaty on TRIPS.

The President of the Associated Chambers of Commerce (ASSOCHAM) said that Basmati rice is traditionally grown in India and granting a patent to it violates the Geographical Indications Act under the TRIPS.

The TRIPS clause defines Geographical indication as “a good originating in the territory of a member, or a region or locality in that territory, where a given quality, reputation, or other characteristic of the goods is essentially attributable to its geographical origin.”

As a result, it is safe to say that Basmati rice is as exclusively associated with India as Champagne is with France and Scotch Whiskey with Scotland.

Indians argue that just as the USA cannot label their wine as Champagne, they should not be able to label their rice as Basmati.

If the patent is not revoked in the USA, because, unlike the Turmeric case, rice growers lack documentation of their traditional skills and knowledge, India may be forced to take the case to the WTO for an authoritative ruling based on violation of the TRIPS.

In the wake of the problems with patents that India has experienced in recent years, it has realized the importance of enacting laws for conserving biodiversity and controlling piracy as well as intellectual property protection legislation that conforms to international laws.

There is a widespread belief that RRT took out a patent on Basmati only because of weak, non-existent Indian laws and the Government’s philosophical attitude that natural products should not be patented.

According to some Indian experts in the field of genetic wealth, India needs to formulate a long-term strategy to protect its bio-resources from future bio-piracy and/or theft.

British traders are also supporting India. According to Howard Jones, marketing controller of the UK’s privately owned distributor Tilda Ltd., “true Basmati can only be grown in India.

We will support them in any way if it’s necessary”. The Middle East is also supported by labeling only Indian rice as Basmati.

Government and government agencies have gathered the necessary data and information to support their case and to prevent their cultural heritage from being taken away from them.

Whether Royal Rice Tec Inc. is guilty of bio-piracy? Explain

Question 1. Discuss whether the decision of the USPTO to grant a patent for the valued Basmati rice violates TRIPS.
Answer: Bio-piracy in general can be defined as the practice of commercially exploiting naturally occurring biochemical or generic material, especially by obtaining patents that restrict its future use, while failing to pay reasonable compensation to the community from which it originates.

It is a manipulation of the Intellectual Property Rights by corporations, entities, and persons to gain exclusive control over the national genetic resources, without giving adequate recognition and remuneration to the original possessors of those resources.

Indigenous people possess significant old knowledge that allowed them to sustainably live and make use of biological arid gone within their natural environment for generations.

Traditional knowledge naturally includes a deep understanding of ecological processes or d e c ability to sustainably extract useful products from the local habitat.

Examples of bio-piracy include the recent patents granted by the Patent and Trademark Office to different American companies or Turmeric’, ‘Neem’ and most notably, ‘Basmati Rice.

All three products have been indigenous to the Indian subcontinent since time immemorial.

RRT’s actions constitute bio-piracy because it infringe the provisions of the Convention on Biological Diversity (‘CBD; in short), which provides for the State’s sovereignty over its genetic resources.

The CBD aims to bring about a system for the conservation and sustainable use of biological diversity and the fair and equitable sharing of benefits arising from the use of their genetic resources.

How RRT established its patent demonstrates that it has ignored the contributions of the local communities in the production of Basrnati and that it does not intend to share the benefits accruing from the use of genetic resources.

This includes both the informal contributions of the farmers who have been growing Basrnati for hundreds of years in India and the neighboring sub-continents, as well as the more formal, scientific breeding work that has been done by rich research institutes to evolve better varieties of Basrnati.

RRT has capitalized on this work of the indigenous community by taking out a Patent on Basrnati and intends to monopolize the commercial use of past research, without giving any recognition or remuneration to those who played a key role in the evolution and breeding of Basrnati rice in its natural habitat.

The involved in the Basrnati patent is therefore classified threefold namely a theft of the collective intellectual and biodiversity heritage of Indian farmers, a theft from Indian traders and exporters whose markets are being stolen by RRT, and finally, a deception of consumers because RRT is using a stolen name Basmati for rice which are derived from Indian rice but not grown in India, and therefore are not the same quality.

RRT has unfairly appropriated and exploited the genetic resources in this case by attempting to gain exclusive control of its development and propagation through a legal process that threatens the traditional rights of the original possessors of the resource.

The key concern relates to RRT’s use of the term ‘Basmati’ to describe its rice lines and grains.

‘Basmati’ is associated with the specific aromatic rice variety grown in India and taking out a Patent on the use of the term to describe its invention.

RRT has potentially reversed the culpability and made India the violator of RRT’s legally protected rights although the latter are the original possessors and breeders of the ‘Basmati’ rice. RRT is guilty of bio-piracy.

Question 2. How does the patent granted to RRT by the USPTO impact the farmers in India? 
Answer: The grant of a Patent to RRT on Basmati does violate certain provisions of TRIPS.

The TRIPS Agreement provides for certain standards to be fulfilled before granting of protection in the form of Intellectual Property Rights which are particularly relevant for determining whether there was any act of bio-piracy involved in the above case.

RRT’s patent on Basmati violates Article 22 of the TRIPS, which deals with Geographical Indications.

As defined under Article 22(1) of TRIPS, Geographical Indications are indications that identify a good as originating in the territory of a member, or a region or locality in that territory, where a given quality, reputation, or other features of the goods is essentially attributable to its geographical origin.

For example, wines and liquors are most commonly associated with Geographical Indications of their place of origin.

The term “Champagne” can only be used to describe a wine that has been produced in the Champagne region of France, the area from which the wine derives its name.

Wine with similar features but produced in another part of the world, cannot be described as “Champagne”.

“Champagne” remains an exclusive product and the name is the exclusive property of the French company producers. A similar case of Geographical Indication is that of “Scotch”, a whisky, which is produced in the Scottish highlands.

This protection for Geographical Indications for wines and liquors is outlined in Article 23 of the TRIPS.

Basmati falls in this category because it enjoys the same closely linked and exclusive relationship with its place of origin in India.

In India, Basmati is grown mainly in some scattered districts of Punjab, Haryana, and Uttar Pradesh. India grows tons of rice annually.

Hence, it is clear that Basmati rice, as it is traditionally recognized, is geographically unique in its origin.

The Basmati Patent resulted in a brief diplomatic crisis between India and the United States with India threatening to take the matter to WTO as an infringement of TRIPs since a Gl product cannot be patented under the provision of TRIPs.

However, ultimately, due to review decisions by the United States Patent Office, RRT has lost most of its claims of the patent, including, most significantly, the right to call their rice “basmati.”

There is a precedent also for the recognition of Basmati as a Geographical Indication by the International Buyers.

The European Commission recognizes India’s and other neighboring sub-continents’ rights to products bearing their distinctive geographical indications, allowing only Basmati rice that has been grown in India and the neighboring sub-continent to be labeled as such.

Similarly, the code of practice for rice in the UK, the largest market for Basmati rice in Europe, describes long grain, aromatic rice grown only in India and neighboring sub-continent as Basmati.

Question 3. Whether adequate legislation exists in India concerning geographical indications? Discuss the salient features. 
Answer:  RRT’s patent could impact Indian farmers in the following two possible ways:

  1. By displacement of Basmati exports from India; and
  2. By monopolizing the Basmati seed supplies.

Regarding the first possible inroads that may be made by the USA into the South Asian export markets, it is a matter of concern to the Indian farmers.

In 1995, the USA produced 7.89 million metric tons of rice and in the same year, India produced 122.37 million metric tons of rice.

American rice exports are significantly greater than India’s, implying that the USA has a greater production surplus. In 1994 itself, the USA exported more volumes of rice as compared to India and its neighboring sub-continent.

Hence, owing to the RRT’s patent, it seems that potential exists for the USA to displace Indian Basmati exports.

Criticism from Indian rice farmers logically ensued, as many were forced to pay royalties to the conglomerate.

The production and cultivation of Basmati has with it a history dating back to centuries ago.

For farmers, the grain is an entity that is constantly evolving.

In the context of India, Basmati rice has always been considered a common resource dependent upon word-of-mouth knowledge and transfer.

Using this logic, Rice Tec alleged that the ‘Basmati name was in the public domain and that by patenting it; they were in actuality protecting its name and origins.

RRT soon came out with hybrid versions of Kasmati, Texmati, and Jasmati, which for rural farmers clearly illustrated the profit-based interest of the conglomerate.

Through its acquisition, RRT patented some 22 varieties of rice.

One of which was Basmati 867, a rice grain that was very similar to the original Basmati but was advertised to have a less chalky more refined taste.

The severity of RRT’s biopiracy cannot be underestimated, as the conglomerate was claiming to have invented the physical characteristics of Basmati such as the plant height and grain length.

By claiming ownership of the rice plant itself, RRT was directly threatening rural farming communities.

A second and more serious threat is that, through its patent, RRT could acquire a monopoly over Basmati seed supply to the sub-continent.

It is a premier developer of commercial hybrid rice varieties in the USA.

A precedent exists that foreign agri-business companies have bought hybrid seeds from Third World Countries.

For instance, Monsanto has recently undertaken a joint venture with Grameen Bank in Bangladesh to distribute its hybrid seeds through loan packages to small farmers.

Hybridization is likely to harm small farmers more as they are less able to absorb the higher seed costs. In its extreme form, such hybridization could harm genetic diversity and deplete farmlands of their intrinsic resources.

Question 5. Explain the provisions for registration of geographical indications in India. 
Answer: In India, the legal system for Geographical Indication (‘Gl’ in short) protection has been developed very recently.

The provisions in that regard are contained in the Geographical Indications of Goods (Registration and Protection) Act (‘Gl Act’ in short) which was enacted in the year 1999 and came into force only in September 2003.

Role Of International Institutions Salient features Of legal Protection To Geographical Indications In India

Available relief include:

  • Injunction,
  • Discovery of documents.
  • Damages or accounts of profits
  • Delivery of the infringing labels and indications for destruction or erasure.

Provisions for the registration on Geographical Indication are as follows:

Section 8 of the Geographical Indications of Goods (Registration & Protection) Act, 1999 provides that a geographical indication may be registered in respect of any or all of the goods, comprised in such class of goods as may be classified by the Registrar and in respect of a definite territory of a country, or a region or locality in that territory, as the case may be.

The Registrar may also classify the goods by the International classification of goods for registration of geographical indications and publish in the prescribed manner in an alphabetical index of classification of goods.

Any question arising as to the class within which any goods fall or the definite area in respect of which the geographical indication is to be registered or where any goods are not specified in the alphabetical index of goods published shall be determined by the Registrar whose decision in the matter shall be final.

Role Of International Institutions Application According to section 11 of the act

Topic Not Yet Asked But Equally Important For Examinations Descriptive Questions

Question 1. Discuss in brief at least five leading International Instruments concerning Intellectual Property Rights.
Answer: Introduction to the leading International Instruments concerning Intellectual Property Rights: Intellectual property has a dual nature, i.e. it has both a national and international dimension.

For instance, patents are governed by national laws and rules of a given country, while international conventions on patents ensure minimum rights and provide certain measures for enforcement of rights by the contracting states.

Strong protection for intellectual property rights (IPR) worldwide is vital to the future economic growth and development of all countries.

Because they create common rules and regulations, international IPR treaties, in turn, are essential to achieving the robust intellectual property protection that spurs global economic expansion and the growth of new technologies.

A list of some leading Instruments concerning Intellectual Property Rights is as below:

  1. The Paris Convention for the Protection of Industrial Property
  2. The Berne Convention for the Protection of Literary and Artistic Works
  3. The WIPO Copyright Treaty (WCT)
  4. The Patent Cooperation Treaty (PCT)
  5. Budapest Treaty on the International Recognition of the Deposit of Microorganisms for Patent Procedure
  6. The Madrid Agreement Concerning the International Registration of Marks and the Protocol Relating to the Madrid Agreement
  7. The Hague Agreement Concerning the International Deposit of Industrial Designs
  8. The Trademark Law Treaty (TLT)
  9. The Patent Law Treaty (PLT)
  10. Treaties on Classification
  11. Special Conventions in the Field of Related Rights: The International Convention for the Protection of Performers, Producers of Phonograms and Broadcasting Organizations (“the Rome Convention”)
  12. Other Special Conventions in the Field of Related Rights
  13. The WIPO Performances and Phonograms Treaty (WPPT)
  14. The International Convention for the Protection of New Varieties of Plants
  15. The Agreement on Trade-Related Aspects of Intellectual Property Rights

Role Of International Institutions International Instrument

Question 2. Write a brief note on the Berne Convention.
Answer: The Berne Convention deals with the protection of works and the rights of their authors.

It is based on three basic principles and contains a series of provisions determining the minimum protection to be granted, as well as special provisions available to developing countries that want to make use of them.

1. The three basic principles are the following:

  1. Works originating in one of the Contracting States (that is, works the author of which is a national of such a State or works first published in such a State) must be given the same protection in each of the other Contracting States as the latter grants to the works of its nationals (principle of “national treatment”).
  2. Protection must not be conditional upon compliance with any formality (principle of “automatic” protection).
  3. Protection is independent of the existence of protection in the country of origin of the work (principle of “independence” of protection).
  4. If, however, a Contracting State provides for a longer term of protection than the minimum prescribed by the Convention and the work ceases to be protected in the country of origin, protection may be denied once protection in the country of origin ceases.

2. The minimum standards of protection relate to the works and rights to be protected, and to the duration of protection:

  • As to works, protection must include “every production in the literary, scientific and artistic domain, whatever the mode or form of its expression” (Article 2(1) of the Convention).
  • Subject to certain allowed reservations, limitations, or exceptions, the following are among the rights that must be recognized as exclusive rights of authorization:
    1. The right to translate
    2. The right to make adaptations and arrangements for the work
    3. The right to perform in public dramatic, dramatico-musical and
      musical works
    4. The right to recite literary works in public
    5. The right to communicate to the public the performance of such works
    6. The right to broadcast (with the possibility that a contracting
    7. The state may provide for a mere right to equitable remuneration instead of a right of authorization)
    8. The right to make reproductions in any manner or form (with the possibility that a Contracting State may permit, in certain special cases, reproduction without authorization, provided that the reproduction does not conflict with the normal exploitation of the work and does not unreasonably prejudice the legitimate interests of the author; and the possibility that a Contracting State may provide, in the case of sound recordings of musical works, for a right to equitable remuneration)
    9. The right to use the work as a basis for an audiovisual work, and the right to reproduce, distribute, perform in public, or communicate to the public that audiovisual work.

The Convention also provides for “moral rights”, that is, the right to claim authorship of the work and the right to object to any mutilation, deformation, or other modification of, or other derogatory action with, the work that would be prejudicial to the author’s honor or reputation.

3. The Berne Convention allows certain limitations and exceptions on economic rights, that is, cases in which protected works may be used without the authorization of the owner of the copyright, and payment of compensation.

These limitations are commonly referred to as “free uses” of protected works and are outlined in Articles 9(2) (reproduction in certain special cases), 10 (quotations and use of works by way of illustration for teaching purposes), 10 bis (reproduction of newspaper or similar articles and use of works to report current events) and 11 bis (3) (ephemeral recordings for broadcasting purposes).

4. The Appendix to (the Paris Act of the Convention also permits developing countries to implement non-voluntary licenses for the translation and reproduction of works in certain cases, in connection with educational activities. In these cases, the described use is allowed without the authorization of the right holder, subject to the payment of remuneration to be fixed by the law.

The Berne Union has an Assembly and an Executive Committee. Every country that is a member of the Union and has adhered to at least the administrative and final provisions of the Stockholm Act is a member of the Assembly.

The members of the Executive Committee are elected from among the members of the Union, except for Switzerland, which is a member ex officio.

Question 3. Write a brief note on the universal copyright convention.
Answer: Universal Copyright Convention: The Universal Copyright Convention (UCC), adopted in Geneva, Switzerland, in 1952, is one of the two principal international conventions protecting copyright; the other is the Berne Convention.

The UCC was developed by the United Nations Educational, Scientific and Cultural Organization (UNESCO) as an alternative to the Berne Convention for those states that disagreed with aspects of the Berne Convention but still wished to participate in some form of multilateral copyright protection.

These states included developing countries as well as the United States and most of Latin America. The former thought that the strong copyright protections granted by the Berne Convention overly benefited Western, developed, copyright-exporting nations, whereas the latter two were already members of the Buenos Aires Convention, a Pan-American copyright convention that was weaker than the Berne Convention.

The Berne Convention states also became a party to the UCC, so that their copyrights would exist in non-Berne convention states. In 1973, the Soviet Union joined the UCC.

Under the Second Protocol of the Universal Copyright Convention (Paris Text), protection under U.S. copyright law is expressly required for works published by the United Nations, by UN specialized agencies, and by the Organization of American States (OAS). The same requirement applies to other contracting states as well.

Berne Convention states were concerned that the existence of the UCC would encourage parties to the Berne Convention to leave that convention and adopt the UCC instead. So the UCC included a clause stating that parties that were also Berne Convention parties need not apply the provisions of the Convention to any former Berne Convention state that renounced the Berne Convention after 1951.

Thus, any state which adopts the Berne Convention is penalized if it then decides to renounce it and use the UCC protections instead, since its copyrights might no longer exist in Berne Convention states.

Since almost all countries are either members or aspiring members of the World Trade Organization (WTO) and are thus conforming to the Agreement on Trade-Related Aspects of Intellectual Property Rights Agreement (TRIPS), the UCC has lost significance.

Question 4. Write a note on the patent cooperation treaty.
Answer:

Patent Co-operation Treaty: The Patent Cooperation Treaty (PCT) is an international patent law treaty, concluded in 1970.

It provides a unified procedure for filing patent applications to protect inventions in each of its contracting states.

A patent application filed under the PCT is called an international application, or PCT application. A single filing of a PCT application is made with a Receiving Office (RO) in one language.

It then results in a search performed by an International Searching Authority (ISA), accompanied by a written opinion regarding the patentability of the invention, which is the subject of the application.

It is optionally followed by a preliminary examination, performed by an International Preliminary Examining Authority (IPEA).

Finally, the relevant national or regional authorities administer matters related to the examination of application (if provided by national law) and issuance of patents.

A PCT application does not itself result in the grant of a patent, since there is no such thing as an “international patent”, and the grant of a patent is a prerogative of each national or regional authority.

In other words, a PCT application, which establishes a filing date in all contracting states, must be followed up with the step of entering into national or regional phases to proceed toward the grant of one or more patents.

The PCT procedure essentially leads to a standard national or regional patent application, which may be granted or rejected according to applicable law, in each jurisdiction in which a patent is desired.

The Patent Cooperation Treaty (PCT) assists applicants in seeking patent protection internationally for their inventions, helps patent Offices with their patent-granting decisions, and facilitates public access to a wealth of technical information relating to those inventions.

By filing one international patent application under the PCT, applicants can simultaneously seek protection for an invention in a very large number of countries.

The contracting states, the states which are parties to the PCT, constitute the International Patent Cooperation Union.

Question 5. What is the WIPO – Development Agenda?
Answer: The World Intellectual Property Organization (WIPO): The World Intellectual Property Organization (WIPO) is one of the 15 specialized agencies of the United Nations (UN). WIPO was created in 1967 “to encourage creative activity, to promote the protection of intellectual property throughout the world”.

WIPO currently has 191 member states, administers 26 international treaties, and is headquartered in Geneva, Switzerland. The current Director-General of WIPO is Francis Gurry, who took office on 1 October 2008.

188 of the UN member states as well as the Cook Islands, Holy See, and Niue are members of WIPO.

Non-members are the states of Federated States of Micronesia, Nauru, Palau, Solomon Islands, and South Sudan. Palestine has permanent observer status.

In October 2004, WIPO agreed to adopt a proposal offered by Argentina and Brazil, the “Proposal for the Establishment of a Development Agenda for WIPO”—from the Geneva Declaration on the Future of the World Intellectual Property Organization. This proposal was well supported by developing countries.

The agreed “WIPO Development Agenda” (composed of over 45 recommendations) was the culmination of a long process of transformation for the organization from one that had historically been primarily aimed at protecting the interests of right holders, to one that has increasingly incorporated the interests of other stakeholders in the international intellectual property system as well as integrating into the broader corpus of international law on human rights, environment, and economic cooperation.

Several civil society bodies have been working on a draft Access to Knowledge (A2K)treaty which they would like to see introduced.

In December 2011, WIPO published its first World Intellectual Property Report on the Changing Face of Innovation, the first such report of the new Office of the Chief Economist. WIPO is also a co-publisher of the Global Innovation Index.

Question 6. Write a note on UNESCO.
Answer:

Copyright a traditional tool for encouraging creativity nowadays, has even greater potential to encourage creativity at the beginning of the 21st century.

Committed to promoting copyright protection since its early days (the Universal Copyright Convention was adopted under UNESCO’s aegis in 1952), UNESCO has over time grown concerned with ensuring general respect for copyright in all fields of creation and cultural industries.

It conducts, in the framework of the Global Alliance for Cultural Diversity, awareness-raising, and capacity-building projects, in addition to information, training, and research in the field of copyright law.

It is particularly involved in developing new initiatives to fight against piracy. The digital revolution has not left copyright protection unaffected.

UNESCO endeavors to contribute to the international debate on this issue, taking into account the development perspective and paying particular attention to the need to maintain a fair balance between the interests of authors and the interest of the general public in access to knowledge and information.

Question 7. Write a brief note on the TRIPS agreement.
Answer: Trade-Related Aspects of Intellectual Property Rights (TRIPS) Agreement: With the establishment of the World Trade Organization (WTO), the importance and role of intellectual property protection has been crystallized in the Trade-Related Intellectual Property Systems (TRIPS) Agreement.

It was negotiated at the end of the Uruguay Round of the General Agreement on Tariffs and Trade (GATT) treaty in 1994.

The general goals of the TRIPS Agreement are contained in the Preamble to the Agreement, which reproduces the basic Uruguay Round negotiating objectives established in the TRIPS area by the 1986 Punta del Este Declaration and the 1 988-89 Mid-Term Review.

These objectives include the reduction of distortions and impediments to international trade, the promotion of effective and adequate protection of intellectual property rights, and ensuring that measures and procedures to enforce intellectual property rights do not themselves become barriers to legitimate trade.

The obligations under TRIPS apply equally to all member states. However, developing countries were allowed extra time to implement the applicable changes to their national laws, in two tiers of transition according to their level of development.

The transition period for developing countries expired in 2005. For least-developed countries, the transition period has been extended to 2016 and could be extended beyond that.

The TRIPS Agreement, which came into effect on 1 January 1 995, is to date the most comprehensive multilateral agreement on intellectual property.

The areas of intellectual property that it covers are:

  1. Copyright and related rights (i.e. the rights of performers, producers of OT sound recordings, and broadcasting organizations)
  2. Trademarks including service marks
  3. Geographical indications including appellations of origin
  4. Industrial designs
  5. Patents including protection of new varieties of plants
  6. The layout designs (topographies) of integrated circuits
  7. The undisclosed information includes trade secrets and test data.

Issues Covered under TRIPS Agreement

The TRIPS agreement broadly focuses on the following issues:

  • How basic principles of the trading system and other international intellectual property agreements should be applied.
  • How to give adequate protection to intellectual property rights.
  • How countries should enforce those rights adequately in their territories.
  • How to settle disputes on intellectual property between members of the WTO.
  • Special transitional agreements during the period when the new system is being introduced.

Features of the Agreement

The main three features of the TRIPS Agreement are as follows Standards:

The TRIPS Agreement sets out the minimum standards of protection to be provided by each Member.

Enforcement: The second main set of provisions deals with domestic procedures and remedies for the enforcement of intellectual property rights.

The Agreement lays down certain general principles applicable to all 1PR enforcement procedures.

Dispute settlement: The Agreement makes disputes between WTO Members about the respect of the TRIPS obligations subject to the WTO’s dispute settlement procedures.

In addition, the Agreement provides for certain basic principles, such as national and most-favored-nation treatment (non-discrimination), and some general rules to ensure that procedural difficulties in acquiring or maintaining IPRs do not nullify the substantive benefits that should flow from the Agreement.

The TRIPS Agreement is a minimum standards agreement, which allows Members to provide more extensive protection of intellectual property if they so wish.

Members are left free to determine the appropriate method of implementing the provisions of the Agreement within their legal system and practice.

Intellectual Property Rights Question And Answers

Intellectual Property Rights Distinguish Between

Question 1. Distinguish between the following:

  1. ‘Intellectual property’ and ‘industrial property’.

Answer:

There are three types of property:

  • Movable property
  • Immovable property
  • See carefully
    1. The term ‘intellectual property’ is coined to indicate the kind of property that covers in it, the creations of the human mind and human intellect.
    2. It consists of valuable information which can be converted into tangible objects
    3. The two types/branches of intellectual property are-
  • Copyright
  • Industrial property
    1. Owners of intellectual property enjoy certain rights like the right to use and licence and certain limitations are also placed upon them.
    2. Intellectual property includes rights relating to
  • Trademarks and Service marks
  • Patents
  • Industrial designs etc.

While

— Industrial property

  • It is a kind of intellectual property.
  • It is a collective name given for rights related to industrial or commercial activities of a person and this reflects industrial or commercial rights.
  • Industrial property includes
    • Patents
    • Trademark, service mark
    • Utility models
    • Industrial design
    • Geographic origin
    • Protection against unfair competition
  • It covers
    • Inventions
    • Creations
    • New products
    • New Processes
    • New design/model
    • Distinct marks

Intellectual Property Rights Descriptive Questions

Question 2. Discuss how labor theory is different from functional theory in justifying intellectual property.
Answer:

Labor theory is also known as positive theory. The underlying principle of the theory is that labor of the individual is the foundation of the property. As per the theory, A product is the property of a person who produces it or brings it into existence.

Property is the result of individual labour and therefore no person has a moral right to the property which has not been acquired by his efforts.

The theory is individualistic in approach in relating to property rights. The functional theory is also known as the sociological theory of property.

According to the theory, no one can be allowed to have an unrestricted use of the property to the detriment of others, the use of the property should conform to the rules of the reason and welfare of the community.

The theory takes into concern the society as a whole and therefore maintains a welfare approach towards the creation of the property.

Question 3. Give your interpretation about the “Proviso” that a person may legitimately acquire property rights by making his labor with resources held “in common” only if, after the acquisition, “there is enough” and as good left in common for others.
Answer:

The Natural Right Theory emanates from the proposition that “a person who labors upon resources that are either un-owned or “held in common” has a natural property right to the fruits of his or her efforts and that the state must respect and enforce that natural right”.

This idea has been elaborated in the writings of John Locke and is also applicable to the subject of intellectual property, wherein the raw materials in the form of facts and concepts do seem in some sense to be “held in common” and where labor contributes substantially to the value of the finished product. Lockean property entitlements.

  1. Right to use without harm
  2. Right to transfer the property
  3. Right of exclusive usage of the property

Personality theory finding place in the writings of Kant and Hegel is that private property rights are crucial to the satisfaction of some fundamental human needs.

The lawmakers thus must create and allocate entitlements to resources in a way that best enables people to satisfy such needs.

From this perspective, Intellectual Property Rights may be justified either on the ground that they shield from appropriation or modification artifacts through which authors and artists have expressed their “wills” (an activity thought central to “personhood”).

Justin Hughes, taking inspiration from Hegel’s Philosophy of Rights, laid down the following guidelines concerning the proper shape of an Intellectual Property regime:

  1. We should be more willing to accord legal protection to the fruits of highly expressive intellectual activities, such as the writing of novels, than to the fruits of less expressive activities, such as genetic research.
  2. Because a person’s “persona” his “public image, including his physical features, mannerisms, and history”  is an important “receptacle for personality.” it deserves generous legal protection, although ordinarily it does not result from labor.
  3. Authors and inventors should be permitted to earn respect, honor, admiration, and money from the public by selling or giving away copies of their works, but should not be permitted to surrender their right to prevent others from mutilating or misattributing their works.

Intellectual property rights afford authors and inventors a measure of control over this risk.

To put the point a different way, it is the moral claims that attach to personality, reputation, and the physical embodiments of these individual goods that justify legal rules covering damage to reputation and certain sorts of economic losses.

Moreover, personality-based theories of intellectual property often appeal to other moral considerations. Hegel’s personality-based justification of intellectual property rights included an incentive-based component as well-he asserts that protecting the sciences promotes them, benefiting society.

Perhaps the best way to protect these intuitively attractive personality-based claims to intangible works is to adopt a more comprehensive system designed to promote progress and social utility.

Question 4. One of the prime factors contributing to the survival and growth of every business in today’s competitive world is the development of intellectual property as an instrument of economic development. Discuss.
What are the major areas in which companies in India can take the lead in the global phenomenon of intellectual property rights?
Answer:

Though the importance of the subject of Intellectual Property Rights has increased with time, it is still not a global phenomenon. The significance and importance accorded to it differs from one country to the other. The chief factors responsible for this difference are:

The amount of resources that different countries allocate and spend towards the creation of intellectual assets; and

The amount of protected knowledge and information that is used in the process of production.

A useful indicator to measure the magnitude of resources devoted towards the creation of new knowledge and information is the country’s expenditure on research and development activities.

Statistics make it clear that Developing countries tend to spend much less on R&D activities as compared to Developed countries.

In general, one of the major factors responsible for the increase in R & D funding is the growing importance and participation of the private sector which has also resulted in an increased reliance on IPR protection by the private players seeking state protection from any encroachment on their intellectual property rights.

This is not only beneficial for such private entities but also for the State since it helps in the creation and dissemination of further knowledge and information in society.

However, it has been observed that such R&D funding by private players is predominant in developed countries only.

The other way in which IPRs influence economic activity in a country is through the use of proprietary knowledge and information — owned by both domestic as well as foreign residents — in both production and consumption.

Data reveals that in countries with relatively low-income generations, the share of agricultural output is comparatively higher than the share of income earned through services.

This also suggests that the IPRs, relating to agricultural processes and products, are more important in developing countries than in developed countries.

The major area in which companies in India can take the lead in the global phenomenon of intellectual property rights is: Agriculture Traditionally, the relevance of Intellectual Property Rights in agricultural research has been very minimal.

This is because Intellectual Property Rights were mostly concentrated towards the protection of the outcome of industries as against agriculture perse.

Furthermore, most of the Research and developments & D) carried out in the agricultural sector were by the public sector institutions in both developed as well as developing countries.

For instance, the development and dissemination of the technology that led to the Green Revolution in India did not pose any substantial conflicts around the subject of IPR. Moreover, there is still reluctance in the developing nations to accept grants of Patents in the agriculture sector.

In India, since the early 1980s, there has been a significant shift in the national policy towards agricultural research.

After having tried and made continuous efforts towards public funding for R&D activities in agriculture, the budget allocation has been rationalized.

At the time, the participation of the private sector in agricultural R&D has grown by metes and bounds. In the developed nations, about one-half of the agricultural R&D is funded by the private sector.

Copyright protection concerning digital content on a worldwide basis same In the 1990s, Copyright protection had gained importance for its role in protecting digital information on the Internet.

The protection of digital content is still not a moot point or an issue in developing countries, where computer and network penetration is much lower as compared to the industrial developed countries.

In early 1998, there were, for example, only 0.2 Internet hosts per 1,000 inhabitants in developing countries compared to 31 in developed countries.

Notwithstanding all this, with a consistent trend towards liberalization of telecommunications services and the plummeting costs of computing and telecommunications technologies, there is an expectation for sustained growth of the Internet in developing countries as well and thus increased relevance of Copyright protection about digital content on a worldwide basis.

Intellectual Property Rights Important For Examinations Descriptive Questions

Question 1. Explain the concept of property. What are the different theories of property?
Answer: The term ‘Property’ has a very wide connotation, it not only includes money and other tangible objects of some value, but it also includes the intangible rights which are considered to be a source or an element of income or wealth.

It includes the rights and interests that a human possesses over land (and chattel) and which is to the exclusion of all others. It is the right to enjoy and dispose of certain things in the way that he pleases, provided that such use is not prohibited by law.

However, there are certain things over which property rights by any single individual (or an entity) cannot be exercised.

This includes the sea, the air, and the like as they cannot be appropriated. Everyone has a right to enjoy them, but no one has an exclusive right over them.

Different theories laid down on the subject of the concept of “Property” are as follows:

  1. Historical Theory of Property
  2. Labour Theory (Spencer)
  3. Psychological Theory (Bentham)
  4. Functional Theory (Jenks, Laski)
  5. Philosophical Theories (Property as a Means to Ethical ends and Property as an End in itself).

According to the Historical theory of Property, the concept of Private Property grew out of joint property.

In the words of Henry Maine, “Private Property was chiefly formed by the gradual disentanglement of the separate rights of the individual from the blended rights of the community”.

In the earlier days, the ownership rights over property were vested in large societies chiefly Patriarchal societies.

However, with the disintegration of societies and families, there was a gradual evolution of the concept of individual rights.

Roscoe Pound in his theory has also pointed out the fact that the earliest form of property was like group property and it was later on when families partitioned that the existence of individual property came to be recognised.

Labour Theory (Spencer)

  • This theory of property is also known as the ‘Positive Theory’. The underlying principle basis of this theory is that the labor of the individuals is the foundation of the property.
  • The theory states that a thing is the property of a person who produces it or brings it into existence.
  • The chief supporter of this theory is Spencer, who developed the theory on the principle of ‘equal freedom’.
  • He stated that property is the result of individual labor, and therefore, no person has a moral right to property that he has not acquired by his effort.

Psychological Theory (Bentham)

  • According to this theory, Property came into existence on account of the acquisitive instinct of human beings. Every individual has a desire to own and have in his possession things which is the factor responsible for bringing Property into existence.
  • According to Bentham, Property is altogether a conception of mind and thus, it is nothing more than an expectation to derive certain advantages from the object according to one’s capacity.
  • Roscoe Pound also supports Bentham on this school of thought and has observed that the sole basis of a conception of Property is the acquisitive instinct of the individual which motivates him to assert his claim over objects in his possession and control.
  • Functional Theory (Jenks and Laski) This theory is also known as the ‘sociological theory of property’, it assumes that the concept of Property should not only be confined to private rights but should be considered as a social institution securing the maximum interests of society.
  • Property is situated in society and has to be used in the society itself. According to Jenks, no one can be allowed to have unrestricted use of his property, to the detriment of others. He thus states that the use of property should conform to the rules of reason and the welfare of the community.
  • According to Laski, who also supports this school of thought, Property is a social fact like any other, and it is the character of social facts to alter.
  • Property has further assumed varied aspects and is capable of changing further with the changing norms of society.

Property is the creation of the State

The origin of ‘Property’ is to be traced back to the origin of ‘Law and ‘State’. Jenks observed that Property and Law were born together and would die together.

It means that Property came into existence when the State framed Laws. As per this theory, Property was non-existent before the Law.

According to Rousseau, ‘it was to convert possession into property and usurpation into a right that Law and State were founded.’

The first who enclosed a piece of land and said- this is mine- was the founder of real society. He insisted on the fact that property is nothing but a systematic expression of degrees and forms of control, use, and enjoyment of things by persons that are recognized and protected by law. Thus, the conclusion is that property was a creation of the State.

Philosophical Theories (Property as a Means to Ethical Ends) In the views of Aristotle, Hegel, and Green, Property has never been treated as an end, but always as a means to some other end.

According to Aristotle, it may be a means to the end of the Good Life of citizens. In the views of Hegal and Green, it may be a means to the fulfillment of the Will without which individuals are not fully human.

According to Rousseau, Jefferson, and Friendman, it may be a means as a pre-requisite of individual freedom seen as a human essence.

Similarly, the outstanding critics of property like Winstanley, and Marx have denounced it as something destructive of human essence, a negative means concerning the ontological end.

Question 2. Explain different connecting points of the Intellectual Property Law regime and Cyber Law regime in India as well as from the international perspective.
Answer: Intellectual Property Rights in the Cyber World: Though both Intellectual Property and the Cyber Law are independent subjects and have their area of operation, however, the influence of one on the other cannot be denied and the same is becoming all the more evident these days.

We live in a world that is dominated by computers and the internet and therefore the world today is appropriately called the Cyber World.

Intellectual Property (IP) equally is an expanding phenomenon with more and more innovations coming to the surface, which also acts as a catalyst for expanding businesses.

However, in terms of vintage and history, Intellectual Property is relatively a longstanding field of legal practice than Cyber Law.

In the Indian context particularly, Cyber Law emerged only with the passage of the Information Technology Act, of 2000, while as an IP law, the Copyright Act was passed in the year 1957.

However, the common theme that runs through both these areas of law is that both have been significantly impacted by the development taking place in the field of technology as well as the growth of the Internet medium.

For instance, a lot of issues cropped up in the area of Copyright law the internet enabled its users to readily reproduce materials available online.

To meet such challenges, the Governments have been, time and again, bringing stringent provisions in the criminal law to catch up with the criminals and thus create deterrence in the society towards such malpractices.

Copyright and Cyberspace – Copyright protection gives the author of the work a certain “bundle of rights”, including the exclusive right to reproduce the work in copies, to prepare derivative works based on the copyrighted work, and to perform or display the work publicly.

Public Performance and Display Rights – The right that does get affected is that of display. Display of the work is also done by making copies, which are then retailed or lent out. This also falls under the right to display, which the holder of the copyright has.

Distribution Rights – Copyright Law grants the holder of the copyright the exclusive right to distribute copies of the work to the public by sale or by the transfer of ownership.

Caching (Mirroring) – It is a violation of the internet. Caching may be local caching and proxy caching.

In addition, proxy caching may give rise to infringement of the right of public distribution, public policy, public performance, and digital performance.

Protection of Database in India – The Indian Copyright Act, 1957 protects “Databases” as “Literary Works” under Section 13(1) (a) of the Act which says that copyright shall subsist throughout India in original literary, dramatic, musical, and artistic works. The term computer Database has been defined in the Information Technology Act, of 2000 for the first time.

Section 43 of the IT Act, 2000 provides for compensation to the aggrieved party up to one Crore rupees from a person who violates the copyright and cyberspace norms. Also, Section 66 of the IT Act, 2000 provides for penal liabilities in such a case.

Internet Protection in India – The internet challenge for the protection of internet is the protection of intellectual property.

It is still unclear as to how copyright law governs or will govern these materials (literary works, pictures, and other creative works) as they appear on the internet. Section 79 of the IT Act 2000 provides for the liability of ISPs “Network Service Providers not to be liable in certain case.”

Section 79 of the IT Act exempts ISPs from liability for third-party information.

Indian Cyber Jurisdiction – Though it is the in nascent stage as of now, jurisprudential development will become essential soon; as the internet and e-commerce shall shrink borders and merge geographical and territorial restrictions on jurisdiction. There are two dimensions to deal with.

  • How foreign courts assume jurisdiction over the internet and relative issues.
  • The consequences of the decree passed by a foreign court. Thus, there is an immense need for Indian society to be made aware of the necessity of copyright protection on all fronts to prevent any unauthorized use and pilferage of the system.

The analysis of copyright in cyberspace reveals a mixed result of new opportunities and threats.

Such threats often outweigh the opportunities offered by cyberspace and the necessity arises for increasing regulations of cyberspace to protect copyrights.

Further lack of internationally agreed principles relating to copyrights in cyberspace gives ample room for divergent domestic standards. Cyberspace – Cyberspace can be described as the virtual world interconnecting human beings through computers and telecommunication without regard to the limitations of physical geography.

With the onset of modern technology, more importantly, the internet, copyright protection has taken a hit and thus the issues relating to it assumed greater significance.

Nowadays, the protections of Copyright law have been extended to protect internet items too. It protects original work or work that is fixed in a tangible medium i.e. it is written, typed, or recorded.

Although the current copyright law protects copyright owners, it has its shortcomings when it comes to its implementation and enforcement on the Cyber World.

Cyberspace is a virtual world, that technically exists only in computer memory, but it is interactive and pulsing with life. IPR and Cyber Space- While the Internet is undoubtedly acclaimed as a major achievement of humankind it cannot be denied that it has come with its own set of challenges.

One of the major challenges that it poses is that it has captured the physical marketplace and has created a new substitute which is the virtual marketplace.

It is thus the responsibility of all IPR owners to protect their IPRs from any mala fide actions of the miscreants operating on the internet medium by invalidating and reducing such mala fide acts attempts of such criminals by taking proactive measures.

It is important to know about the copyright issues associated with computer programs/software, computer databases, and various other works in cyberspace.

Under the TRIPS (Trade-Related Aspects of Intellectual Property Rights) agreement, Computer Programs also now qualify for Copyright protection just as any other literary work is afforded.

Question 3. Trace the common features of Intellectual Property Rights and Human Rights.
Answer: Intellectual Property Rights as Human Rights: Human Rights and Intellectual Property, though two very different sets of laws with no apparent connection, have gradually become intimate bedfellows. Since its inception, the two subjects developed virtually in isolation from each other.

But in the last few years, international standard-setting activities have begun to map previously uncharted intersections between intellectual property law on the one hand and human rights law on the other.

Exactly how this new-found relationship will evolve is being actively studied – and sometimes even fought over – by states and non-governmental organizations (NGOs) in international venues such as the World Intellectual Property Organization (WIPO), the U.N.

Commission on Human Rights and the Sub-Commission on the Promotion and Protection of Human Rights, the World Trade Organization (WTO), the World Health Organization (WHO), and the Conference of the Parties to the Convention on Biological Diversity (CBD).

A look at the lawmaking that is underway in these for a prima facie reveals two distinct conceptual approaches to the interface between human rights and intellectual property. The first approach finds that there is a conflict between human rights and intellectual property rights.

This view believes that a regime of strong intellectual property protection undermines and therefore is incompatible with human, rights obligations, especially in the area of economic, social, and cultural rights.

To resolve this conflict it is suggested the normative privacy of human rights law over intellectual property law should be recognized in areas where specific treaty obligations conflict.

In the second approach, Human Rights and Intellectual Property Rights are seen as concerned with the same fundamental question, i.e., defining the appropriate scope of private monopoly power that gives authors and inventors a sufficient incentive to create and innovate, while ensuring that the consuming public has adequate access to the fruits of their efforts.

This school of thought sees Human Rights law and the Intellectual Property Rights law as essentially compatible, although often disagreeing over where to strike the balance between incentives on the one hand and access on the other.

But the principal reason for the execution of these agreements lies not in deontological claims about inalienable liberties, but rather in the economic and instrumental benefits that flow from protecting Intellectual Property products across national borders.

It is also true that both areas of law were preoccupied with more important issues, and neither saw the other as either aiding or threatening its sphere of influence or opportunities for expansion.

This evolutionary process resulted in a de facto separation of human rights into categories, ranging from a core set of peremptory norms for the most egregious forms of state misconduct to civil and political rights, to economic, social, and cultural rights.

Among these categories, economic, social, and cultural rights are the least well-developed and the least prescriptive, having received significant jurisprudential attention only in the last decade.

Human Rights law added little to these two enterprises. It provided neither a necessity nor sufficient justification for demanding strong, state-granted intellectual property monopolies (whether bundled with trade rules or not). Nor, conversely, did it function as a potential check on the expansion of Intellectual Property Law.

Question 4. List a few advantages of including the process of evaluation of Intellectual Property in an Organisation. What are the different methods adopted for such evaluation?
Answer: Advantages of Intellectual Property Valuation: Intellectual Property Rights, such as Patents and Trademarks, which require necessary registration with the concerned authority, provide legal evidence of one’s ownership over such intangible assets while also ensuring one’s peaceful and exclusive right to the use of such property.

It gives one the right to exclude others from the use of such rights. This means that one is armed with the legal remedy against any infringement by the competitor(s).

Moreover, it is also an asset that can be profitably licensed or sold to others to provide them with the rights they would otherwise not have, and consequent to these benefits an increase in the total value of one’s business.

At times, the process of valuation of an enterprise’s Intellectual Property itself requires registration of such property as a precondition which enables the process of monetizing one’s intangible assets.

Conducting a valuation of one’s Intellectual Property has its significance and advantages.

For instance, assessing the value of one’s Patent, Trademark, or Copyright may simplify the licensing or assignment process, and help one to determine the royalty rates that should be paid as a result of using one’s intellectual property assets.

Further, ascribing a reasonable valuation to one’s intellectual property, if not currently accounted for, increases the overall value of one’s business and provides one with collateral for loans and mortgages.

The difference between quantitative and qualitative valuations: An intellectual property can be valued on various parameters, but the overarching principle guiding the valuation process is, how much of a competitive advantage one’s intellectual property provides over others in the industry. While evaluating the worth of intellectual property, the following two methods of valuation have traditionally been used:

Quantitative valuation: As the name itself suggests, this method relies on measurable data or numerical information to produce an estimate of the value of one’s intangible assets. It attempts to answer the question by providing a monetary value or contribution that the intellectual property provides, whether directly to the business or indirectly by increasing the value of other parts of the operation or the appeal to investors.

Qualitative methods: The parameters of valuation under this method are very different from the quantitative valuation method. This method provides a non-monetary estimate of the value of an Intellectual Property by rating it based on its strategic impact, loyalty held by consumers, its impact on the company’s future growth, and other intangible metrics that do not rely solely on numbers.

These two kinds of valuation methods should however not be presumed to be contradictory or mutually exclusive; depending on the needs of one’s business, one may employ a variation of methods that fall into both categories.

It would not be wrong to suggest that the two valuation methods are perhaps the two sides of the same coin. Quantitative and qualitative attempts to tackle the question of firm value from different viewpoints, which may both come in useful depending on the audience in question and the reason for valuation.

In conclusion, it can be said that determining the value of one’s Intellectual property can be a very challenging task and an exhausting process.

But obtaining a valuation can result in significant benefits to one’s business and thus the need for valuation can neither be underestimated nor be undermined.

Following the valuation models described above to break down the process into simple steps and establish a clear purpose and audience for the valuation, can make valuation manageable.

Question 5. What are the different competing rationales put forward for the protection of Intellectual Property?
Answer: Competing Rationales for the Protection of Intellectual Property Rights:

  • Intellectual property indeed is now one of the valuable assets in commercial transactions, be it intellectual property licensing, joint ventures, foreign collaborations, manufacturing, purchase or distribution agreements, or mergers and acquisitions.
  • Licenses to use patents, copyrights, and trademarks, are often combined with transfers of know-how and are increasingly an important term in technology transactions.
  • These licenses provide royalty revenues to the owner of the Intellectual Property and distribute products and technologies to licensees who might not otherwise have had access to them.
  • In such transactions, the licensees may also gain rights to create improvements or derivative works and to develop their own Intellectual Property assets, which can then be cross-licensed or licensed to others. This creates a very productive cycle of innovation and invention and adds to the revenues of the companies.
  • Intellectual property laws confer the right to own intellectual assets by its creator and also enable him to make profits from his artistic, scientific, and technological creations for a defined period.
  • Such rights apply to the intellectual creations and not the physical object in which it is embodied. Countries have enacted laws to protect intellectual property for two main reasons.
  • One is to give statutory expression to the moral and economic rights of creators in their creations and the rights of the public to access such creations.
  • The second is to promote creativity and its dissemination which results in economic and social development.

The World Intellectual Property Report 2011: The Changing Face of Innovation new WIPO publication describes how ownership of intellectual property (IP) rights has become central to the strategies of innovating firms worldwide.

  • With global demand for patents rising from 800,000 applications in the early 1980s to 1.8 million in 2009, the Report concludes that growing investments in innovation and the globalization of economic activities are key drivers of this trend.
  • As a result, IP policy has moved to the forefront of innovation policy WIPO Director General, Francis Gurry, notes that “innovation growth is no longer the prerogative of high-income countries alone; the technological gap between richer and poorer countries is narrowing. Incremental and more local forms of innovation contribute to economic and social development, on a par with world-class technological innovations.”
  • Intellectual property assets are used not only in business transactions but are also traded in their own right such as online exchanges for the evaluation, buying, selling, and licensing of patents and other forms of Intellectual Property.
  • The buyers and sellers of intellectual property manage their intellectual property as financial assets just as investors in stocks, options, and other financial instruments.
  • Strong intellectual property rights help consumers make an educated choice about the safety, reliability, and effectiveness of their purchases.
  • Enforced intellectual property rights ensure products are authentic, and of the high quality that consumers recognize and expect. IP rights foster the confidence and ease of mind that consumers’ demands and markets rely on.

Question 6. What are “intellectual property rights”? List out the subject matter protected by IPR under the World Intellectual Property Organization.
Answer:

Intellect means perception. It is the barometer of one’s understanding of persons or things of events and concepts, individually or collectively.

Intellectual Property (IP) refers to the creations of the human mind like inventions, literary and artistic works, and symbols, names, images, and designs used in commerce. Intellectual property is divided into two parts:

Industrial property and Copyright. Industrial property includes inventions (patents), trademarks, industrial designs, and geographic indications of source and Copyright includes literary and artistic works such as novels, poems and plays, films, musical works, artistic works such as drawings, paintings, sculptures, and architectural designs.

Intellectual property encompasses four separate and distinct types of intangible property namely:

  1. Patents,
  2. Trademarks,
  3. Copyrights,
  4. Trade secrets.

All the above types of intangible property collectively are referred to as “intellectual property”.

Products that used to be traded as low-technology goods or commodities now contain a higher proportion of invention and design in their value.

For example, brand-named clothing. Therefore, creators are given the right to prevent others from using their inventions, designs, or other creations. These rights are known as “intellectual property rights”.

The convention establishing the World Intellectual Property Organization (1967) listed the subject matter protected by intellectual property rights as follows:

  1. literary, artistic, and scientific works;
  2. Performances of performing artists, phonograms, and broadcasts
  3. Inventions in all fields of human endeavor;
  4. Scientific discoveries;
  5. Industrial designs;
  6. Trademarks, service marks, commercial names and designations;
  7. “All other rights resulting from intellectual activity in the industrial, scientific, literary or artistic fields” and
  8. Protection against unfair competition.

Patents Question And Answers

Patents Act Question and Answers

Question 1. Write a note on the ‘patent co-operation treaty’.
Answer: The Patent Co-operation Treaty (PCT) is a multilateral treaty that became effective in 1978. The PCT is administered by the International Bureau of the World Intellectual Property Organization (WIPO) whose headquarters is in Geneva, Switzerland.

The member countries of the PCT are called PCT Contracting States. As of August 1, 2006, there were 133 PCT Contracting States.

The PCT enables a patent application to file one “international” patent application to seek protection in any or all of the PCT Contracting States. Patents are granted or rejected by each PCT Contracting State or regional officer individually under their respective patent laws.

Thus, an applicant must still prosecute a patent application in each country or regional officer in which he seeks protection and pay the national or regional fees.

The main advantage of filing a PCT application is the additional time gained before having to prosecute applications in other countries after the initial filing.

Without the PCT the applicant generally has 12 months to file patent applications in other Paris Convention countries after filing the initial application in contrast, by using the PCT the application has at least 30 months (and more in many countries) from the date of initial filing to begin prosecuting his application in other countries effectively gaining 18 months.

This delay provides time to obtain knowledge as to the patentability and commercial prospects of an invention.

It also postpones the major costs of internationalizing a patent application such as paying national/regional fees, translating the patent application, and paying fees to local patent agents in various countries. The PCT procedure consists of two main phases; the “international phase” and the “national phase”.

The international phase consists of:

  1. Filing of the international application either with a national/regional “Receiving Office” or the International Bureau of WIPO
  2. Novelty search on the patentability of the invention (including an international search report and a written opinion on potential patentability)
  3. Publication of both the PCT application and the international search report by W1PO, and (Optional step) request for an international preliminary examination of the international application.

National Phase

After the international phase, the application enters the “national” phase, which consists of processing the international application before each Contracting State that has been designated in the international application and in which the applicant wishes to pursue patent protection.

Certain requirements must be fulfilled to enter the national phase.

These requirements include paying national fees and if necessary, furnishing a translation of the application (as filed and/or amended).

Note that the filing of the PCT request together with the application constitutes the designation of all Contracting States that are bound by the Treaty on the international filing date. In the national phase, the applicant selects the particular States in which he wishes to obtain protection for his invention.

A PCT application must contain the following elements: request, description, one or more claims, one or more drawings (where drawings are necessary for the understanding of the invention), and an abstract.

The request is simply a form that is filed with the international application. Any national or resident of one of the PCT Contracting States may file an international patent application.

Patents Descriptive Question and Answers

Question 1. Briefly discusses the Paris Convention for the Protection of Industrial Property.
Answer: Paris Convention for the Protection of Industrial Property: The Paris Union, established by the Convention, has an Assembly and an Executive Committee.

Every State member of the Union who has adhered to at least the administrative and final provisions of the Stockholm Act (1967) is a member of the Assembly.

The members of the Executive Committee are elected from among the members of the Union, except for Switzerland, which is a member ex officio.

The Paris Convention concluded in 1883, was revised at Brussels in 1900, at Washington in 1911, at The Hague in 1925, at London in 1934, at Lisbon in 1958, and at Stockholm in 1967, and it was amended in 1979.

The Convention applies to industrial property in the widest sense, including patents, marks, industrial designs, utility models, trade names, geographical indications, and the repression of unfair competition.

The substantive provisions of the Convention may be divided into three main categories namely national treatment, right of priority, and common rules.

Question 2. What are the advantages of PCT filing?
Answer:

Advantages of PCT Filing:

The advantages of PCT filing for the applicant, the patent offices, and the general public are given below:

  1. The applicant has up to 1 8 months more than in a procedure outside the PCT to reflect on the desirability of seeking protection in foreign countries
  2. To appoint local patent agents in each foreign country
  3. To prepare the necessary translations and to pay the national fees. The PCT filing assures the applicant that if his international application is in the form prescribed by the PCT, it cannot be rejected on formal grounds by any designated Office during the national phase of the processing of the application.
  4. Based on the international search report, the applicant can evaluate with reasonable probability the chances of his invention being patented. Based on the international preliminary examination report, that probability is even stronger; the applicant can amend the international application to put it in order before processing by the designated Offices.

Question 3. Briefly discuss the Berne convention for the protection of literary and Artistic work.
Answer: Berne Convention for the Protection of Literary and Artistic Works: The Berne Union has an Assembly and an Executive Committee. Every country member of the Union that has adhered to

at least the administrative and final provisions of the Stockholm Act is a member of the Assembly. The members of the Executive Committee are elected from among the members of the Union, except for Switzerland, which is a member ex officio.

The Berne Convention concluded in 1886, was revised at Paris in 1896 and at Berlin in 1908, completed at Berne in 1914, revised at Rome in 1928, at Brussels in 1948, at Stockholm in 1967, and at Paris in 1971, and was amended in 1979.

The Convention rests on three basic principles and contains a series of provisions determining the minimum protection to be granted, as well as special provisions available to developing countries.

Basic Principles The three basic principles are the following:

  1. Works originating in one of the contracting States must be given the same protection in each of the other contracting States as the latter grants to the works of its nationals.
  2. Such protection must not be conditional upon compliance with any formality
  3. Such protection is independent of the existence of protection in the country of origin of the work.
  4. If, however, a contracting State provides for a longer term than the minimum prescribed by the Convention and the work ceases to be protected in the country of origin, protection may be denied once protection in the country of origin ceases.

The minimum standards of protection relate to the works and rights to be protected, and the duration of the protection:

As to works, the protection must include every production in the literary, scientific, and artistic domain, whatever may be the mode or form of its expression.

Subject to certain permitted reservations, limitations, or exceptions, the following are among the rights that must be recognized as exclusive rights of authorization:

  1. The right to translate
  2. The right to make adaptations and arrangements of the work,
  3. The right to perform in public dramatic, dramatico-musical and
    musical works
  4. The right to recite in public literary works
  5. The right to communicate to the public the performance of such
    works
  6. The right to broadcast (with the possibility of a contracting state to provide for a mere right to equitable remuneration instead of a right of authorization)
  7. The Right To Make Reproductions In Any Manner Or Form.
  8. The right to use the work as a basis for an audiovisual work, and the right to reproduce, distribute, perform in public, or communicate to the public that audiovisual work.

Question 4. Explain salient features of the Patents (Amendment) Act, 2002.
Answer: The second phase of the amendment was brought in by the Patents (Amendment) Act, 2002 which came into force on 20th May 2003

The main features of the amendments included:

  1. The term of a patent was extended from 14 to 20 years, wherein the date of the patent was the date of filing of the complete specification. Also, the difference in terms of a drug/food patent and another patent was removed.
  2. The definition of “invention” was made in conformity with the provisions of the TRIPS Agreement by introducing the concept of inventive step, thereby enlarging the scope of invention.
  3. A deferred examination system was introduced.
  4. Introduction of the provision of publication of application after 18 months from the date of filing thereby bringing India at par with the rest of the world.
  5. Microorganisms became patentable, whereas inventions relating to traditional knowledge were included in the list of “what are not inventions”.
  6. The concept of unity of invention under EPC and PCT.
  7. Section 39 was reintroduced thereby prohibiting the Indian residents from applying abroad without prior permission or first filing in India.
  8. Provisions of Appellate Board were brought in by inserting section 11 6. All appeals to the decision of the Controller would be appealable before the Appellate Board. The headquarters of the Appellate Board is to be in Chennai.
  9. Section 117 provided for Bolar’s provision for the benefit of the agrochemical and pharmaceutical industry.
  10. The third and final amendment to the Patents Act, 1970 came by way of the Patents (Amendment) Ordinance, 2004, which was later replaced by The Patent (Amendment) Act, 2005, and Patents (Amendment) Rules, 2006 with retrospective effect from 1st January 2005. With the third amendment, India met with the international obligations under the TRIPS. Significant achievements of this amendment were:

Deletion of section 5. opening of the mailbox and granting of product patents. Thus this amendment led to the dawn of the “product patent regime” in India.

Abolition of Exclusive Marketing Rights (EMR).

The definition of “startup” under rule 2(FB) has been substituted with a new definition. A more liberal definition of startup has been incorporated that can allow domestic as well as foreign entities to claim benefits such as fast-track mechanisms and lower fees for filing patents. According to the Patent (Amendment) Rules, 2017-

Patents (Amendment) Act, 2002 “Startup” means

  1. An entity in India recognized as a startup by the competent authority under the Startup India Initiative.
  2. In the case of a foreign entity, an entity fulfilling the criteria for turnover and period of incorporation/ registration as per Startup India Initiative and submitting a declaration to that effect.

Explanation: In calculating the turnover, reference rates of foreign currency of the Reserve Bank of India shall prevail.

Patents (Amendment) Act, 2002 Current Position

The present Indian position in respect of patent law is governed by the provisions of the Patents Act, 1 970 as amended by the Patents (Amendment) Act, 2005 (hereinafter referred to as the Act) and Patents (Amendment) Rules, 2017 (hereinafter referred to as the Rules) Department of Industrial Policy and Promotion (DIPP) has amended Patent Rules 2003 with effect from 1s’ December 201 7 called as the Patent (Amendment) Rules, 2017.

  • The Head Patent Office is located in Kolkata and its branch offices are located in Delhi, Mumbai, and Chennai.
  • The patent system in India is administered by the Controller General of Patents, Designs, Trademarks, and Geographical Indications.
  • Each office has its territorial jurisdiction for receiving patent applications and is empowered to deal with all sections of the Patent Act.

Question 5. Discuss the ways, through which CBD and TRIPs could be harmonized.
Answer: Harmonization of CBD and TRIPs

The Convention on Biological Diversity (CBD) 1992: Opened for signature at the Earth Summit in Rio de Janeiro in 1992, and entering into force in December 1993, the Convention on Biological Diversity is an international treaty for the conservation of biodiversity, the sustainable use of the components of biodiversity and the equitable sharing of the benefits derived from the use of genetic resources.

The interface between biodiversity and intellectual property is shaped at the international level by several treaties and processes, including at the WIPO, and the TRIPS Council of the WTO.

With 193 Parties, the Convention has near universal participation among countries. The Convention seeks to address all threats to biodiversity and ecosystem services, including threats from climate change, through scientific assessments, the development of tools, incentives, and processes, the transfer of technologies and good practices, and the full and active involvement of relevant stakeholders including indigenous and local communities, youth, NGOs, women, and the business community.

The Cartagena Protocol on Biosafety is a subsidiary agreement to the Convention. It seeks to protect biological diversity from the potential risks posed by living modified organisms resulting from modern biotechnology.

The treaty defines biodiversity as “the variability among living organisms from all sources including, inter alia, terrestrial, marine, and other aquatic ecosystems and the ecological complexes of which they are part; this includes diversity within species, between species, and of ecosystems.”

The Convention on Biological Diversity establishes important principles regarding the protection of biodiversity while recognizing the vast commercial value of the planet’s store of germplasm.

However, the expansion of international trade agreements establishing a global regime of intellectual property rights creates incentives that may destroy biodiversity, while undercutting social and economic development opportunities as well as cultural diversity. The member countries were pressured to change their IPR laws to conform to the TRIPS agreement.

While the TRIPS and the CBD both attempt to legislate some form of intellectual property and technology transfer, the Agreement appear to provide contradictory prescriptions for the control over genetic control over genetic resources and biodiversity.

The two Agreements embody and promote conflicting objectives, systems of rights, and obligations. The core issues are that, in the area of patentable subject matter, benefit sharing, protection of local knowledge, requirements of prior informed consent, and the role of the state.

Major tension between the CBD and the TRIPS is related to the case of National Sovereignty and the Rights of IPR Holders. Through the CBD, countries have the right to regulate access of foreigners to biological resources and knowledge and to determine benefit-sharing arrangements.

The TRIPS enables persons or institutions to patent a country’s biological resources in countries outside the country of origin of the resources or knowledge.

In this manner, TRIPS facilitates the conditions for misappropriation of ownership or rights over living organisms, knowledge, and processes on the use of biodiversity.

In the benefit-sharing arrangements, a key aspect of the CBD is the one, which recognizes the sovereign rights of the states over their biodiversity and knowledge, and thus gives the State the right to regulate access and this in turn, enables the state to enforce its rights on arrangements for sharing benefits.

Access where granted, shall be on mutually agreed terms (Art 1 5.4) and shall be subject to prior informed consent (Art 15.6).

Most importantly, each country shall take legislative, administrative, or policy measures with the aim of sharing fairly and equitably the results of research and development and the benefits arising from the commercial and other utilization of genetic resources with the contracting party providing such resources. Such sharing shall be upon mutually agreed terms.

The TRIPS is a device with an international intellectual property regime that maximizes the potential for both traditional knowledge and modern scientific innovations to contribute to economic progress. To achieve this goal, the TRIPS needs to be reviewed and incorporated further:

  • Establish the concept of community property rights for Traditional Knowledge recognition;
  • Recognize communities’ rights over their resources and TK;
  • Recognizes safeguards and protect the TK, innovations, practices, and technologies of indigenous and local people and communities;
  • Mandate legal protection for TK;
  • Recognize the sovereign rights of states over their biodiversity and genetic resources;
  • Mandate the principles of prior informed consent and benefit sharing when other countries access the biogenetic resources and local communities.

Question 6. Does a patent obtained in India give protection worldwide? Is it possible to file an international application in India under the Patent ) Cooperation Treaty (PCT)?
Answer:

  1. A patent obtained in India does not provide protection worldwide.
  2. Patent protection is effective only within the territory of India.
  3. Separate patents should be obtained in each country where the applicant requires protection for his invention in those countries.
  4. It is possible to file an international application in India under the Patent Co-operation Treaty (PCT) in the patent offices located at Kolkata, Chennai, Mumbai & Delhi.

Patents Practical Questions

Question 1. Read the Novartis case on the patenting law of Gleevec and answer the questions that follow:
Answer:

Novartis vs. Union of India & Others is a landmark decision by a two-judge bench of the Supreme Court of India on the issue of whether Novartis could Patent Gleevec in India, and was the culmination of a seven-year-long litigation fought by Novartis. The Supreme Court upheld the Indian Patent Office’s rejection of the patent application.

The patent application claimed the final form of Gleevec (the beta-crystalline form of imatinib mesylate). In 1993, during the time India did not allow patents on products, Novartis had patented imatinib, with salts vaguely specified, in many countries but could not patent it in India.

The key differences between the two patent applications, were that the 1998 patent application specified the counterion (Gleevec is a specific salt imatinib mesylate) while the 1 993 patent application did not claim any specific salts nor did it mention mesylate, and the 1998 patent application specified the solid form of Gleevec the way the individual molecules are packed together into a solid when the drug itself is manufactured (this is separate from processes by which the drug itself is formulated into pills or capsules) while the 1993 patent application did not. The solid form of imatinib mesylate in Gleevec is beta crystalline.

In 2000, the United States Food and Drug Administration (FDA) approved imatinib mesylate in its beta crystalline form, sold by Novartis as Gleevec (U.S.) or Glivec (Europe/Australia/Latin America).

TIME magazine hailed Gleevec in 2001 as the ‘magic bullet’ to cure cancer. Both Novartis patents on the freebase form of imatinib and the beta crystalline form of imatinib mesylate are listed by Novartis in the FDA’s Orange Book entry for Gleevec. As provided under the TRIPS agreement, Novartis applied for exclusive marketing rights (EMR) for Gleevec from the Indian Patent Office, and the EMR was granted in November 2003.

Novartis made use of the EMR to obtain orders against some generic manufacturers who had already launched Gleevec in India. Novartis set the price of Gleevec at USD 2,666 per patient per month; while the generic companies were selling their versions at USD 177 to 266 per patient per month.

Novartis also initiated a program to assist patients who could not afford its version of the drug, concurrent with its product launch. The Intellectual Property Appellate Board (IPAB) was formed and in 2007 the case was transferred before the IPAB in line with section 117G of the Patents Act, 1970.

The IPAB on 26,h June 2009 modified the decision of the Assistant Controller of Patents and Designs stating that ingredients for grant of patent novelty and non-obviousness to the person skilled in the art were present in the application but rejected the application on the ground that the drug is not a new substance but an amended version of a known compound and that Novartis was unable to show any significant increase in the efficacy of the drug and it, therefore, failed the test laid down by section 3(d) of the Patents Act, 1970.

When the examination of Novartis’ patent application began in 2005, it came under immediate attack from opposition initiated by generic companies that were already selling Gleevec in India and by advocacy groups.

The application was rejected by the Patent Office and by an Appeal Board.

The key basis for the rejection was the part of Indian patent law that was created by an amendment in 2005, describing the patentability of new uses for known drugs and modifications of known drugs.

That section, Paragraph 3d, specified that such inventions are patentable only if “they differ significantly in properties concerning efficacy.” At one point, Novartis went to court to try to invalidate Paragraph 3d; it argued that the provision was unconstitutionally vague and that it violated TRIPS.

Novartis lost that case and did not appeal. However, Novartis did appeal the rejection by the Patent Office to India’s Supreme Court, which took the case. The Supreme Court case hinged on the interpretation of Paragraph 3d.

The Supreme Court decided that the substance that Novartis sought to patent was indeed a modification of a known drug (the raw form of imatinib, which was publicly disclosed in the 1993 patent application and scientific articles), that Novartis did not present evidence of a difference in therapeutic efficacy between the final form of Gleevec and the raw form of imatinib, and that therefore the patent application was properly rejected by the patent office and lower courts.

Although the court ruled narrowly and took care to note that the subject application was filed during a time of transition in Indian patent law, the decision generated widespread global news coverage and reignited debates on balancing public good with monopolistic pricing and innovation with affordability.

Had Novartis won and gotten its patent issued, it could not have prevented generic companies in India from continuing to sell generic Gleevec, but it could have obligated them to pay a reasonable royalty under a ‘grandfather clause’ included in India’s patent law.

Question 2. Why did Novartis file the case in the Supreme Court only after India signed
Answer: India accepted product patents as part of the World Trade Organisation (WTO) deal hence Gleevec patent could be registered and enforced by the Indian Courts.

  1. The patent application at the center of the case was filed by Novartis in India in 1998 after India had agreed to enter the World Trade Organization and abide by worldwide intellectual property standards under the TRIPS agreement.
  2. As part of this agreement, India made changes to its patent law; the biggest of which was that before these changes, patents on products were not allowed, while afterward they were, albeit with restrictions.
  3. These changes came into effect in 2005, so Novartis’s patent application waited in a “mailbox” with others until then, under procedures that India Instituted to manage the transition.
  4. India also passed certain amendments to its Patent Law in 2005, just before the laws came into effect, which played a key role in the rejection of the patent application.

Question3. Gleevec’s patent has already been granted in 45 other countries including China. What will the Indian industry gain/lose in the rejection of the patent in India?
Answer: 

Indian industry gains in the rejection of the patents:

  1. Savings in outward remittance of foreign exchange
  2. Dumping shall be restricted
  3. Generic Medicines shall be available at cheaper rates
  4. Growth of Indian Pharma Companies
  5. Enhancement of innovation by Indian Pharma Companies

Indian industry losses in the rejection of the patents:

  1. Multinational Companies will invest less money in research in India
  2. Hinders Medical progress
  3. Indian Industry will lose credibility
  4. Multinational Companies will not do R&D in India
  5. Better Technology transfer from outside is not possible

Question 3. What is your opinion on Novartis’ claim that the beta crystalline packing
Answer: A novel invention is one, that has not been disclosed, in the prior art where prior art means everything that has been published, presented, or otherwise disclosed to the public including documents in foreign languages disclosed in any format in any country of the world on the date of the patent? For an invention to be judged as a novel, the disclosed information should not be available in the ‘prior art’.

The “beta crystalline form” of the molecule is a specific polymorph of imatinib mesylate; a specific way that the individual molecules pack together to form a solid.

This is the actual form of the drug sold as Gleevec; a salt (imatinib mesylate) as opposed to a free base, and the beta crystalline form as opposed to the alpha or other form.

So, by going through the concept of novelty, the process of “beta crystalline packing in solid form” passes the test of novelty, since, this process is not disclosed anywhere in the prior art.

But, if anything is to be patentable, then the sole test of novelty is not sufficient. By Section 3 (d) (as amended), we also have to test whether the same differs significantly in properties about efficacy.

Section 3 (d) of the Indian Patent Act, 1970 (as amended) reads as follows: ‘The mere discovery of a new form of a known substance which does not result in the enhancement of the known efficacy of that substance or the mere discovery of any new property or new use for a known substance or of the mere use of a known process, machine or apparatus unless such known process results in a new product or employs at least one new reactant.

As the beta crystalline form of Imatinib Mesylate is a pharmaceutical substance and a polymorph of Imatinib Mesylate, it directly runs into explanation to Section 3 (d) of the Act As Novartis was unable to show any significant increase in the efficacy of the drugs, hence it failed in the test laid down by explanation to Section 3 (d) of the Act. So, the same is not patentable under the Indian Patent Act, of 1970.

Question 4. What do you understand by the ‘grandfather clause’ of the Novartis patent developed when India did not have product patents?
Answer: Section 11 A (7) of The Patents Act, 1970 provides that on or from the date of publication of the patent application and until the date of grant of a patent in respect of such application, the applicant shall have the like privileges and rights as if a patent for invention had been granted on the date of publication of the application.

However, the applicant shall have no right to institute any proceeding for infringement until the patent has been granted.

Additionally, the rights of a patentee in respect of applications made under section 5 (2) of the Patents Act before January 1, 2005, shall accrue from the date of grant of patent.

Moreover, after the patent is granted in respect of applications made under section 5 (2), the patent holder shall only be entitled to receive reasonable royalty from such enterprises which have made significant investments and were producing and marketing the concerned product before January 1, 2005, and which continue to manufacture the product covered by the patent on the date of grant of the patent and no infringement proceedings shall be instituted against such enterprises.

The above provision is termed a “grandfather clause” in common parlance.

The above grandfather clause created “a special regime for generic versions of medicines if the initial patent application was made between the 1st of January, 1995 and the 31st of December, 2004 and if these medicines were already on the Indian market before the 1st of January, 2005.

Generics that enter into this category can stay on the Indian market even if their pharmaceutical substance is patented.

However, the Indian Law requires that the producers of those generics then pay a “reasonable royalty” to the patent holder.

If Novartis won the case and got the patent, then also the Indian Companies could have continued to sell generic Gleevec, but they have to enter a grandfather clause with Novartis and shall be obligated to pay a reasonable royalty to the patent holder.